You are on page 1of 28

Krok 2- 2019 (General Medicine) (t.

me/krok2GeneralMedicine) - 1

5. A 6-month-old child on breastfeeding is hospitalized in


Krok 2 – 2019 (General Medicine) the inpatient department. After the child recovers, the
doctor recommends the mother to start introducing solid
food to the child‟s diet. What products should be
1. After a pain attack in the right subcostal area, a 58- introduced to the child‟s diet first?
year-old woman with overnutrition developed icteric skin 1. Buckwheat porridge
and sclera, light colored feces, and dark urine. Her 2. Fermented dairy products
abdomen is distended and painful on palpation in the 3. Grated apple
right subcostal area. Palpation detects liver enlarged by 4. Semolina porridge
2-3 cm. Blood test: Total bilirubin – 90 mcmol/L, 5. Vegetable puree
conjugated bilirubic – 60 mcmol/L. What method of
examination will be most informative for diagnosis 6. A 52-year-old man for the last 3 years has been
clarification? suffering from difficult swallowing of solid food, burning
1. Infusion cholegraphy retrosternal pain that aggravated during eating, loss of
2. US of the hepatopancreatobiliary zone body mass and occasional vomiting with undigested
3. Retrograde cholangiopancreatography food. Esophageal X-ray shows S-shaped deformation of
4. Percutaneous transhepatic cholegraphy the esophagus and its dilation; at the cardiac orifice the
5. Intravenous cholegraphy esophagus is constricted; Esophageal mucosa is
smooth, without signs of peristalsis. Make the provisional
2. A 20-year-old student after failing an exam developed diagnosis:
complaints of a sensation of a round foreign body in her 1. Esophageal diverticulum
throat, difficult swallowing. She fixates on her condition, 2. Diaphragmatic hernia
limits her diet, often cries, seeks attention, exhibits 3. Reflux esophagitis
demonstrative attitude. She is highly susceptible to 4. Esophageal achalasia
psychotherapeutic suggestion. What psychiatric 5. Esophageal carcinoma
diagnosis can be made in this case?
1. Hypochondriacal neurosis 7. Disease onset was acute. A child developed general
2. Hysterical neurosis weakness, pain in the joints and elevated temperature.
3. Paranoid personality disorder Later these signs became accompanied by itching skin
4. Depressive neurosis rash manifested as erythematous spots 2-5 mm in size.
5. Obsessive neurosis The rash gradually turned hemorrhagic. Large joints are
painful and swollen; pain attacks periodically occur in the
3. A 32-year-old woman complaints of episodes of paraumbilical area; there are signs of intestinal
intense fear that occur without visible cause and last for haemorrhage. What is the most likely diagnosis?
10-20 minutes; the episodes are characterized by raped 1. Scarlet fever
pulse, sweating, labored breathing and vertigo. Specify 2. Hemorrhagic meningoencephalitis
the likely diagnosis: 3. Streptococcal impetigo
1. Simple schizophrenia 4. Rheumatism
2. Paranoid syndrome 5. Hemorrhagic vasculitis (Henoch-Schonlein purpura)
3. Panic disorder
4. Manic syndrome 8. 2 hours after a traffic accident a 28 year old man in a
5. Claustrophobia grave condition was brought to a hospital. The patient
complains of abdominal pain. He received a blow to the
4. A 48-year-old woman developed insomnia, depressive abdomen wit the steering wheel. Objective examination
mood, anxiety, fears and suicidal thoughts after the revealed the following: the abdomen does not participate
death of her husband that occurred one month ago. in respiration, is tense and acutely painful on palpation;
During her stay in the hospital she speaks in a low voice, the abdominal muscles are defensively tense, peritoneal
is depressed, anxious, avoids sleeping, refuses to eat. irritation signs are positive, hepatic dullness is absent.
What medications should be prescribed in this case? BP is 90/60 mmHg, heart rate is 120/min. What further
1. Antidepressants treatment tactics should be chosen?
2. Group B vitamins 1. Ultrasound investigation
3. Antipsychotics 2. Laparoscopy
4. Anticonvulsants 3. Laparotomy
5. Nootropics
Krok 2- 2019 (General Medicine) (t.me/krok2GeneralMedicine) - 2

4. Cold to the abdomen 5. 12-week pregnancy, threatened spontaneous


5. Laparocentesis abortion

9. Employees work in conditions of high dust 13. A 32-year-old woman complains of body weight loss
concentration. Certain chemical (silicon dioxide content) despite her increased appetite, nervousness, and tremor
and physical properties of dust aerosols contribute to the if the extremities. Objectively: the skin is moist; the
development of occupational dust induced diseases. thyroid gland is diffusely enlarged, painless, soft and
What is the main physical property of dust aerosols? mobile. Blood test: increased level of T3, T4 and thyroid-
1. Solubility stimulating hormone(THS). What is the most likely
2. Ionization diagnosis?
3. Electric charge 1. Thyroid carcinoma
4. Magnetization 2. Diffuse nontoxic goiter
5. Dispersion 3. Autoimmune(Hashimoto‟s) thyroiditis
4. Diffuse toxic goiter
10. A 37-year-old patient complains of pain in the spinal 5. Thyroid adenoma
column, reduced mobility. The condition persists for 7
years. „Sway back‟ is observed, there is no movement in 14. A 22-day old infant developed subcutaneous red
all spinal regions. X-ray shows „bamboo spine‟ vertebral nodes from 1.0 to 1.5 cm in size on the scalp; later the
column. What is the most likely diagnosis? nodes suppurated. Temperature increased upto 37.7 oC,
1. Osteochondrosis intoxication symptoms appeared, regional lymph nodes
2. Tuberculous spondylitis enlarged. Complete blood count: anemia, leucocytosis,
3. Spondylolisthesis neutrocytosis, increased ESR. What diagnosis can be
4. Spondylitis deformans made?
5. Ankylosing spondyloarthritis 1. Vesiculopustulosis
2. Pemphigus
11. During regular medical examination a lyceum 3. Pseudofurunculosis
student presents with signs of cheilitis that manifests as 4. Scalp phlegmon
epithelial maceration in the area of lip seal. The lips are
bright-red, with single vertical cracks covered with 15. A man was brought into the admission room after an
brown-red scabs. These clinical signs are most likely overexposure to cold. He complains of sharp pain in the
caused by insufficient content of the following in this diet: small of his back and elevated body temperature upto 38
1. Calciferol o He took some aspirin. Blood test: leukocyutes – 10.5 x
2. Thiamine 1012/L, eosinophils – 5%, band neutrophils – 8%,
3. Retinol segmented neutrophils – 51%, lymphocytes – 32%,
4. Riboflavin monocytes – 4%, erythrocyte sedimentation rate – 28
5. Ascorbic acid mm/hour. Urinalysis: Protein – 0.6 g/L, leukocytes –
cover the whole vision field, large amount of mucus.
12. A 25-year-old woman was brought into gynecological What is the most likely diagnosis?
department with profuse bloody discharge from her 1. Acute glomerulonephritis
genital tracts. She is 12 weeks pregnant; the pregnancy 2. Chronic pyelonephritis
is planned. Within the last 3 days she was experiencing 3. Acute pyelonephritis
pains in her lower abdomen that eventually started 4. Subacute malignant glomerulonephritis
resembling cramps, she developed bleeding. Her skin is 5. Tubulointerstitial nephritis
pale, pulse- 88/min., blood pressure- 100/60 mm Hg,
body temperature – 36.8°C . Vaginal examination: the 16. A 47-year-old man developed the signs of
uterus size corresponds with 11 weeks of pregnancy, the decompensated laryngeal stenosis against the
cervical canal allows inserting 1 finger and contains background of acute phlegmonous laryngitis. He
fragments of the fertilized ovum, the discharge is bloody presents with inspiratory dyspnea at rest, forced
and profuse. What is the most likely diagnosis? position, cyanotic skin covered in cold sweat,
1. 12-week pregnancy, spontaneous abortion in tachycardia, deficient pulse, and low blood pressure.
progress What urgent treatment tactics should be chosen?
2. Disturbed menstrual cycle, hyperpoylme-norrhea 1. Oxygen therapy
3. Disturbed menstrual cycle, amenorrhea 2. Oral administration of hypersensitization substances
4. Full-term pregnancy, term labor and broncholytics
Krok 2- 2019 (General Medicine) (t.me/krok2GeneralMedicine) - 3

3. Administration of glucocorticoid hormones occurred in the patient?


4. Tracheostomy 1. Gastritis
5. Intravenous administration of dehydrating agents 2. Migraine
3. Somatoform autonomic dysfunction of hypotonic type
17. A 45-year-old man underwent a cardiac surgery one 4. Posthemorrhagic anaemia
week ago. His general state has been deteriorating since 5. Dysmenorrhea
then; dyspnea at rest, retrosternal pain that irradiates to
the neck, marked weakness. Objectively his body 21. After eating shrimps, a 25 year old man suddenly
temperature is hectic. His cardiac borders are expanded, developed skin itching, some areas of his skin became
apical beat is weakened. Auscultation detects pericardial hyperemic or erupted into vesicles. Make the diagnosis:
friction rub. What is the most likely diagnosis? 1. Psoriasis
1. Pulmonary embolism 2. Acute urticaria
2. Myocardial infarction 3. Urticaria pigmentosa
3. Acute pericarditis 4. Scabies
4. Acute myogenic dilation of the heart 5. Hemorrhagic vasculitis (Henoch-Schonlein purpura)
5. Acute cardiac aneurysm
22. A 35-year-old man complains of marked dyspnea
18. An 11 year old boy for a month has been presenting and cardiac pain. He describes his disease to the case
with increasing pain in the right femur. In the painful area of influenza that he had 2 weeks ago. Objectively he
there is a non mobile painful tumor with unclear margins. leans forward when sitting. The face is swollen, cyanotic,
The child complains of general indisposition, weakness, cervical veins are distended. Heart borders are extended
increased body temperature up to 39oC. X-ray shows on both sides, heart sounds are muffled, heart rate = Ps
widened medullary cavity, small foci of cancellous bone = 118/min, BP is 90/60 mmHg. Blood test: ESR is 46
destruction, and onion like lamellar exfoliation of the mm/hour. ECG shows low voltage. X-ray shows
cortical layer. What is the most likely pathology resulting trapezoidal cardiac silhouette and signs of pulmonary
in such clinical presentation? congestion. Choose the treatment tactics:
1. Ewing sarcoma 1. Antibiotics
2. Chondrosarcoma 2. Pericardectomy
3. Juxtacortical sarcoma 3. Pericardial puncture (pericardiocenthesis)
4. Fibrosarcoma 4. Diuretics
5. Osteogenic sarcoma 5. Glucocorticosteriods

19. After a surgery for a left thigh phlegmon the disease 23. A 7-year-old boy has been an inpatient for 1.5
progression was complicate by sepsis. On the 7th day months. He had been brought to the hospital with
after the surgery, there are marked signs of a complaints of edemas all over his body, low urine output
generalized inflammatory reaction, in blood there are and headache. Clinical urinalysis: proteins – 7.1 g/L,
signs of toxic anemia and progressing hypoprotieinemia, leukocytes – 1-2 in the vision field, erythrocytes – 3-4 in
bilirubin levels are 40 mcmol/L, AST and ALT exceed the the vision field. During the course of treatment, the
norm by 2.5 times. Oliguria persists (700 mL of urine per edemas gradually dissipated, headache abated, diuresis
day). Name the phase of sepsis progression: normalized. Daily urine proteins – 3g/L. Biochemical
1. Stress phase blood test; total protein – 43.2 g/L, urea – 5.2 mmol/L,
2. Recovery phase cholesterol – 9.2 mmol/L. What glomerulonephritis
3. Mixed phase syndrome is the most likely to be present in the patient?
4. Catabolic phase 1. Mixed
5. Anabolic phase 2. Nephritic
3. Isolated urinary
20. A 14-year-old girl came to the general practitioner 4. Hematuric
with complaints of weakness, loss of appetite, headache, 5. Nephrotic
rapid fatigability. Her last menstruation was profuse and
lasted for 14 days after the previous delay of 2 months. 24. A 3-year-old child has been brought to a hospital
Objectively; the skin is pale, heart rate is 90/min, BP is with complaints of pain in the legs, fever, and loss of
110/70 mmHg, Hb is 88 g/L. Rectal examination; the appetite. Objectively: pale skin and mucosa,
uterus and its appendages are without changes, no haemorrhagic rash. Lymph nodes are enlarged,
discharge from the genital tracts. What complication painless, dense and elastic, not matted together. Bones,
Krok 2- 2019 (General Medicine) (t.me/krok2GeneralMedicine) - 4

joints, and abdomen are painful. The liver and spleen 2. Paroxysmal supraventricular tachycardia
are enlarged. Hemogram: Hb- 88 g/L, color index – 1.3, 3. Respiratory arrhythmia
platelets – 80 x 109/L, leukocytes – 25.8 x 109/L, 4. Atrial fibrillation
lymphoblasts – 70% , ESR- 52mm/hour. Make the 5. Paroxysmal ventricular tachycardia
provisional diagnosis:
1. Acute rheumatic fever 28. A 62-year-old patient has been hospitalized with
2. Thrombocytopenic purpura complaints of pain in the thorax on the right during
3. Acute leukaemia breathing, dyspnea and dry cough. Ten days ago he
4. Infectious mononucleosis slipped and fell hitting his right side. On examination: the
5. Haemorrhagic vasculitis( Henoch-Schonlein purpura) patient lies on the left side. The right side of the thorax
lags during breathing. On the right there are crepitation
25. A 43-year-old man complains of a protrusion in the and pain in the III-IV ribs. Dullness of percussion sound
right inguinal region, that enlarges due to strain. He has and sharply diminished breath sounds can be observed.
been presenting with this condition for 6 months. Within On X-ray; signs of exudate, fracture of the III-IV ribs. On
this period the protrusion has grown. Objectively in the pleurocentesis; blood is detected. Choose the further
right inguinal region an elastic protrusion 8×5 cm is tactics:
visible. On palpation it disappears, leaving an empty 1. Perform repeated pleural taps
space 4×4 cm between the pedicles of the Poupart 2. Transfer to a thoracic surgery department
ligament. “Cough push” sign is positive over this 3. Prescribe conservative therapy
opening. Make the diagnosis: 4. Apply a fixation bandage to the rib cage
1. Cyst of the right spermatic cord 5. Refer to a traumatologist
2. Right-sided reducible inguinal hernia
3. Right-sided reducible femoral hernia 29. A woman in her early to mild thirties has lost her
4. Right-sided reducible arcuate line hernia consciousness 3-5 minutes ago. On examination: the
5. Right-sided inguinal lymphadenitis skin is pale, no pulse over the carotid arteries, no
spontaneous respiration, pupils are dilated; the patient is
26. A 32-year-old woman complains of tumor like nonresponsive, presents with atony. The patient‟s
formation on the anterior surface of her neck that condition can be determined as:
appeared 2 year ago. Within the last 3 months the tumor 1. Syncope
has been rapidly growing. It hinders swallowing and 2. Brain death
impairs speech; the tumor causes a sensation of 3. Comatose state
pressure. Objectively the skin moisture is normal. Pulse 4. Natural death
is 80/min, rhythmic, blood pressure is 130/80 mmHg. In 5. Clinical death
the right lobe of the thyroid gland there is a dense lumpy
node 3.0×3.5 cm that moves during swallowing. 30. A 9-month-old infant presents with delayed tooth
Scanning image shows a „cold nodule‟ in the thyroid eruption and fontanel closure, weakness and excessive
gland. Make the provisional diagnosis: sweating. What type of hypovitaminosis is the most likely
1. Thyroid cancer in this child?
2. Thyroid cyst 1. Hypovitaminosis B6
3. Thyroid adenoma 2. Hypovitaminosis A
4. Nodular goiter 3. Hypovitaminosis D
5. Autoimmune thyroiditis 4. Hypovitaminosis B1
5. Hypovitaminosis C
27. A 38-year-old woman after physical overexertion
suddenly developed palpitations, dyspnea and dull pain 31. A 45-year-old woman complains of intolerable
in the cardiac area. For 10 years she has been paroxysmal facial pain on the left with attacks that last
registered for regular check-ups due to rheumatism and for 1-2 minutes. Attacks are provoked by chewing. The
mitral valve disease with non-distributed blood disease onset was two months ago after the
circulation. Objectively her pulse is 96/min of unequal overexposure to cold. Objectively: pain at the exit points
strength. Blood pressure is 110/70 mmHg, heart rate is of the trigeminal nerve on the left. Touching near the
120/min. ECG registers small unevenly sized waves in wing of the nose on the left induces a pain attack with
place of P waves, R-R intervals are of unequal length. tonic spasm of the facial muscles. What is the most likely
What is the most likely diagnosis? diagnosis?
1. Atrial flutter 1. Facial migraine
Krok 2- 2019 (General Medicine) (t.me/krok2GeneralMedicine) - 5

2. Trigeminal neuralgia 1. Daily proteinuria over 3.0


3. Temporomandibular joint arthritis 2. Gross hematuria
4. Glossopharyngeal neuralgia 3. Daily proteinuria under 3.0
5. Maxillary sinusitis 4. Increased blood creatinine and blood urea
5. Leukocyturia gross hematuria
32. During analysis of morbidity in the city, it was
determined that age structure of population is different in 36. A 38-year-old woman works in flax processing, she
each district. What statistical method allows to exclude dries flax. She came to the hospital complaining of
this factor, so that it would not skew the morbidity data? difficult breathing, constricting sensation in her chest and
1. Standardization cough attacks. These signs appear on the first day of her
2. Dynamic time series analysis working week and gradually diminish on the following
3. Correlation regression analysis days. What respiratory disease is likely in this case?
4. Wilcoxon signed rank test 1. Silicosis
5. Analysis of average values 2. Bronchial asthma
3. Allergic rhinopharyngitis
33. A 72-year-old man with pneumonia complains of 4. Byssinosis
marked dyspnea, chest pain, severe cough with 5. Asthmatic bronchitis
expectoration, temperature is 39.5-40.0 oC, no urination
for a whole day. Objectively the patient is conscious. 37. A 33-year-old man developed multiple rashes on the
Respiratory rate is 36/min. Over the right lower skin of his torso and extensor surfaces of his upper and
pulmonary lobe percussion sound is dull; on auscultation lower limbs. The rashes itch and occasionally fuse
there is bronchial respiration and numerous moist together and form plaques. The elements of rash are
crackles. BP is 80/60 mmHg. Heart rate is 120/min. covered with silver-white fine scales that easily flake off
Heart sounds are muffled, there is tachycardia. What when scratched. Grattage test results in three sequential
tactics should the family doctor choose in the phenomena: Stearin spot, terminal film and punctate
management of this patient? haemorrhage. What diagnosis can be suspected?
1. Outpatient treatment 1. Pyoderma
2. Hospitalization into the pulmonology unit 2. Lichen ruber planus
3. Hospitalization into the neurology unit 3. Parapsoriasis
4. Hospitalization into the intensive care unit 4. Secondary popular syphilid
5. Treatment in the day patient facility 5. Psoriasis

34. A 59-year-old woman was brought into the 38. On the 15th day after a small trauma of the right foot,
rheumatology unit. Extremely severe case of the patient developed indisposition, fatigability, irritability,
scleroderma is suspected. Objectively she presents with headache, elevated body temperature, and sensation of
malnourishment “mask-like” face, and acro-osteolysis. constriction, tension and twitching in the muscles of the
Blood: erythrocytes – 2.2 x 109 /L, erythrocyte right shin. What disease can be suspected?
sedimentation rate – 40 mm/hour. Urine: elevated levels 1. Anaerobic gas gangrene
of free oxyproline. Name one of the most likely 2. Tetanus
pathogenetic links in this case: 3. Thrombophlebitis of the popliteal artery
1. Formation of antibodies to collagen 4. Acute thrombophlebitis
2. Formation of antibodies to blood corpuscles 5. Erysipelas
3. Formation of antibodies to vessel wall
4. Formation of antibodies to transversely striated 39. A 27-year-old man complains of pain in his leg joints,
muscles purulent discharge from the eyes and painful burning
5. Formation of antibodies to native DNA sensations during urination. Disease onset was acute.
He has a history of influenza. The patient smokes and
35. After overexposure to cold a 45 year old woman drinks alcohol in excess. In his line of work he is often
developed acute pain in her suprapubic and lumbar away on business trips. What is the most likely
areas during urination, sharp pains at the end of etiological factor of this disease?
urination, false urges to urinate. Urine is turbid with 1. Candida
blood streaks. The doctor suspects urinary tract 2. Adenovirus
infection. What results of laboratory analysis would be 3. Staphylococci
the most indicative of such infection?
Krok 2- 2019 (General Medicine) (t.me/krok2GeneralMedicine) - 6

4. Chlamydia lesser pelvis. What is the most likely diagnosis?


5. Streptococci 1. Hyperactive uterine contractions
2. Abruption of normally positioned placenta
40. A 53-year-old man complains of general weakness, 3. Complete uterine rupture
loss of appetite, a painful vesicles appearing on his skin. 4. Threatened uterine rupture
The disease onset occurred suddenly, after 5. Anatomically contracted pelvis
hyperinsolation one week ago. Examination detects
isolated vesicles with wrinkled opercula and occasional 44. A district doctor has diagnosed one of his patients
painful erosions on the skin of the patient‟s torso and with dysentery. What accounting document reflects this
limbs. Nikolsky sign is positive. What is the most likely type of morbidity?
diagnosis? 1. Urgent report
1. Toxicodermia 2. Certificate of temporary disability
2. Acantholytic pemphigus 3. Report on a major non-epidemic disease
3. Herpes 4. Statistical report
4. Duhring‟s disease (Dermatitis herpetiformis) 5. Control card of a patient registered for regular check-
5. Nonacantholytic pemphigus ups

41. A 40 year old man, a welder, uses manganese 45. A 57-year-old patient complains of dyspnea at rest.
electrodes in his line of work (18 year of experience). He The patient presents with orthopnoea, acrocyanosis,
complains of difficulties with writing, bad mood, bulging cervical veins. On percussion: dull sound over
inertness, gait abnormalities, problems with speech, and the lower lung segments. On auscultation: no respiratory
hand tremors. Objectively the following is observed in sounds. Heart rate is 92/min. Right-sided cardiac
the patient: hypomimia, increased muscle tone of plastic dilatation is observed. The liver is +7cm. Shins are
type, and quiet monotonous speech, tremor of the swollen. Pleural effusion is suspected. What indicator
tongue, pill-rolling tremor of the fingers, and retropulsion. would confirm the presence of transudate in this case?
What syndrome developed in this patient due to 1. Positive Rivalta‟s test
manganese poisoning? 2. Total protein content in the pleural fluid below 25 g/L
1. Polyneuritic syndrome 3. Total protein content in the pleural fluid exceeding
2. Meningism 30g/L
3. Parkinsonism 4. Specific gravity exceeding 1015
4. Vestibular syndrome 5. Presence of atypical cells
5. Hypothalamic syndrome
46. A 40-year man claims that his wife is cheating on
42. A 42 year old man, a dispatcher, suffers from peptic him and presents a „proof‟ of her infidelity. He repeatedly
ulcer disease of the duodenum. The disease is of initiated scandals with his wife at home and at work,
moderate severity. He wants to be assigned a disability demanding that she confess her infidelity, insulted her
group. Make the conclusion regarding his working ability: and threatened to kill her. What preventive measures
1. Capable of working, non-employable should be taken against socially dangerous actions on
2. First group of disability his part?
3. Capable of working, employable 1. Family counselling
4. Second group of disability 2. Consultation with the psychologist
5. Third group of disability 3. Consultation with the psychiatrist
4. Consultation with the general practitioner
43. A woman is 40 weeks pregnant. The fetus is in the 5. Outpatient treatment
longitudinal lie and cephalic presentation. Pelvic size:
26-29-31-20. Expected weight of the fetus is 4800 gram. 47. A woman with the pregnancy term of 8 weeks
The labor contractions has been lasting for 12 hours, complains of elevated temperature upto 37.6°C, skin
within the last 2 hours they were extremely painful, the rash that can be characterized as macular exanthema,
parturient woman is anxious. The waters broke 4 hours enlargement of posterior cervical and occipital lymph
ago. On external examination the contraction ring is nodes, small amount of bloody discharge from the
located 2 finger widths above the navel, Henkel-Vasten genital tracts. She was examined by the infectious
sign is positive. Fetal heart rate is 160/min, muffled. On diseases specialist and diagnosed with rubella. What
internal examination the uterine cervix is fully open, the tactics should the obstetrician-gynaecologist choose?
head is engaged and pressed to the entrance into the 1. Treatment of incipient abortion
Krok 2- 2019 (General Medicine) (t.me/krok2GeneralMedicine) - 7

2. Prescription of hemostatic therapy 2. Neisseria meningitis


3. Prescription of antiviral therapy 3. Mumps virus
4. Prescription of antibacterial therapy 4. Streptococcus
5. Abortion 5. Rubella virus

48. A 26 year old woman is suspected to suffer from 52. A 45-year-old man diagnosed with acute pulmonary
systemic lupus erythematosus due to systemic lesions of abscess suddenly developed sharp pain in his chest on
skin, vessels, joints, serous tunics and heart that the right and dyspnea upto 30/min. Examination detects
developed after photosensitization. The following is facial cyanosis and shallow rapid respirations.
detected: LE cells, antibodies to native DNA, isolated Auscultation reveals acutely weakened respiration
anti-centromere antibodies, rheumatoid factor is 1:100, throughout the whole right lung; percussion reveals a
Wassermann reaction is positive, circulating immune vesiculotympanititc (bandbox) resonance at the lung
complex is 120 units. What immunological indicators are apex and dullness in the lower lobe. What complication
considered to be specific to this disease? developed in this patient?
1. Anti centromere antibodies 1. Pneumothorax
2. Immunoglobulin A 2. Pyopneumothorax
3. DNA antibodies 3. Pleuropneumonia
4. Increased circulating immune complex 4. Esophageal perforation
5. Rheumatoid factor 5. Acute mediasinitis

49. A patient has the second and third degree burns of 53. A 30-year-old maltigravida has been in labour for 18
the 15% of the body surface. On the 20th day after the hours. 2 hour ago the pushing stage began. Fetal heart
trauma the patient presents with sharp increase of body rate is clear, rhythmic, 136/min. Vaginal examination
presents with sharp increase of body temperature, reveals complete cervical dilation, the fetal head in the
general weakness, rapid vesicular respiration; facial pelvic outlet plane. Sagittal suture is in line with obstetric
features are sharpened, BP is 90/50 mmHg, heart rate is conjugate, the occipital fontanel is near the pubis. The
112/min. What complication is it? patient has been diagnosed with primary uterine inertia.
1. Acute intoxication What is the further tactics of labor management?
2. Pneumonia 1. Skin head Ivanov‟s forceps
3. Sepsis 2. Labour stimulation
4. Purulent bronchitis 3. Outlet forceps
5. Anaerobic infection 4. Cesarean section
5. Vacuum extraction of the fetus
50. A 45 year old woman came to the maternity clinic
with complaints of periodical pains in her mammary 54. A 17-year-old girl has made an appointment with
glands that start 1 day before menstruation and stop doctor. She plans to begin her sex life. No signs of
after the menstruation begins. Palpation of the mammary gynecological pathology were detected. In the family
glands detects diffuse nodes predominantly in the upper history the patient‟s grandmother had cervical cancer.
outer quadrants. What is the most likely diagnosis? The patient was consulted about the maintenance of her
1. Breast cyst reproductive health. What recommendation will be most
2. Hyperprolactinemia helpful for prevention of invasive cervical cancer?
3. Fibrocystic mastopathy 1. Vitamins, calcium, omega 3
4. Breast cancer 2. Antiviral and antibacterial drugs
5. Mastitis 3. Timely treatment of sexually transmitted diseases
4. Immunomodulators
51. A 26-year-old man complains of chills, rhinitis, dry 5. Vaccination against human papillomavirus (HPV)
cough and fever up to 38 o Examination shows him to be
in a moderately severe condition; there are small pale 55. A 1-year old child with a case of URTI suddenly
pink non-merging spots on the skin of his back, developed noisy respirations with difficult inspiration,
abdomen and extremities. Palpation reveals enlarged intercostal retractions and barking cough on the 2nd
occipital and axillary lymph nodes. No information about night after the disease onset. What is the most likely
vaccination history could be obtained. What is the likely diagnosis?
etiology of this disease? 1. Bronchial asthma
1. Epstein-barr virus 2. Acute bronchiolitis
Krok 2- 2019 (General Medicine) (t.me/krok2GeneralMedicine) - 8

3. Acute bronchitis his fingers and toes. When he returned home, his
4. Acute pulmonary inflammation parents noticed that the tips of his fingers and toes were
5. Stenosing laryngotracheobronchitis white and their sensitivity was lost. As the affected areas
were warming up, the fingers and toes developed
56. A 35 year old man complains of rapidly increasing tingling and painful sensations. Skin pallor changed into
fatigue, palpitations, „visual snow‟, and dizziness. He has redness, tingling stopped, mild itching and swelling of
a history of peptic ulcer of the stomach. Objectively the fingers appeared. Determine the frostbite degree in this
skin is pale. Vesicular respiration is observed in the child:
lungs. Systolic murmur is detected over the cardiac 1. Frostbite of the III degree
apex, heart rate is 100/min., BP is 100/70 mmHg. The 2. Frostbite of the I degree
epigastrium is slightly tender on palpation. Blood test: 3. Frostbite of the II degree
erythrocytes – 3.2 x 1012/L, Hb – 100 g/L, color index – 4. Frostbite of the IV degree
0.95. What type of anemia is the most likely present in 5. Perniosis
this case?
1. Hypoplastic anemia 60. A 72 year old man complains of lower extremity
2. Post-haemorrhagic anemia edema, sensation of heaviness in the right subcostal
3. Chronic iron-deficiency anemia area, dyspnea at rest. For over 25 years he has been
4. Hemolytic anemia suffering from COPD. Objectively: Orthopnea, jugular
5. Sideroblastic anemia venous distension, diffuse cyanosis, acrocyanosis.
Barrel chest is observed, on „percussion there is a
57. A 40-year-old man with Bekhterev disease vesiculotympanitic (bandbox) resonance, sharply
(ankylosing spondylitis) complains of elevated body weakened vesicular respiration on both sides, moist
temperature upto 37.8°C, back pain and stiffness, crepitant crackles in the lower segments of the lungs.
especially observed during the second half of the night. Heart sounds are weakened, the II heart sound is
This condition has been lasting for 2 years. Objectively: accentuated over the pulmonary artery. The liver is +3
reduced spinal mobility, painful sacroiliac joint. cm. What complicated the clinical course of COPD in
erythrocyte sedimentation rate – 45 mm/hour. X-ray this patient?
shows narrowing of the intervertebral disc space and of 1. Pulmonary embolism
the sacroiliac joint. What eye pathology is often 2. Community acquired pneumonia
associated with this type of disease progression ? 3. Chronic pulmonary heart
1. Cataract 4. Acute left ventricular failure
2. Blepharitis 5. Diffuse pneumosclerosis
3. Retinal detachment
4. Optic nerve atrophy 61. A woman has been provisionally diagnosed with
5. Iridocyclitis pheochromocytoma. At the stage of intermission her BP
is within norm; there is tendency towards tachycardia.
58. A child is 1 year old. After solid food was introduced No urine pathologies. The decision has been made to
into the diet, within the last several months the child perform a provocative test with histamine. What drug
developed loss of appetite, diarrhea with large amount of should be kept close at hand for emergency aid in case
feces, and occasional vomiting. Body temperature of positive test result?
remains normal. Body weight is 7 kg. The child is very 1. Phentolamine
pale, has leg edemas and extremely distended 2. Prednisolone
abdomen. Feces analysis detects high levels of fatty 3. Pipolphen (Promethazine)
acids and soaps. Diagnosis of celiac disease was made 4. Mesaton (Phenylephrine)
and gluten free diet was prescribed. What should be 5. Nifedipine
excluded from the diet in this case?
1. Easily digestible carbohydrates 62. Clinical trials have proved the „Lipoflacon‟ drug to be
2. Cereals – Wheat, oats effective for treatment of unstable angina pectoris in the
3. Animal protein control group and experimental group of patients.
4. Fruits Neither patients nor researchers knew who belonged to
5. Milk and dairy products which group. Name this type of study:
1. Multicenter study
59. A 10-year-old boy, who was outdoors in windy and 2. Simple blind study
cold weather, developed moderate pain and tingling in 3. Triple blind study
Krok 2- 2019 (General Medicine) (t.me/krok2GeneralMedicine) - 9

4. Total blind study 67. A 20 year old man, a calibrator of dosimetry


5. Double blind study equipment, committed a gross violation of safety
regulations, when he put two ampoules with cobal-60,
63. Human body receives from the atmosphere a each with the radioactivity of 7 MCi, in the pockets of his
number of chemicals. What type of action results in the trousers and jacket. He has been keeping the ampoules
combined effects that is less than the sum of isolated in his pocket for 8 hours. The tissue at the distance of
effects of these chemicals on the body? 0.5 cm from the source received the dose of 30 Gy
1. Complex action (3000 R), while the tissues at the distance of 20 cm – 2
2. Isolated action R. Did this man develop radiation sickness?
3. Synergistic action 1. Yes, he developed a severe form of acute radiation
4. Potentiation syndrome
5. Antagonism 2. Yes, he developed a mild form of acute radiation
syndrome
64. An 18-year-old young man complains of pain in his 3. Yes, he developed chronic radiation syndrome
knee and elbow joints and body temperature upto 39.5 o 4. Yes, he developed a moderate form of acute
One week and a half earlier he developed sore throat. radiation syndrome
On examination his body temperature is 38.5 oC. 5. No, he did not
Swelling of the knee and elbow joints is observed. Pulse
is 106/min, rhythmic. Blood pressure is 90/60 mmHg. 68. A 45-year-old man, a farmer, presents with acute
Cardiac borders are unchanged, heart sounds are onset of a disease. He complains of headache, high
weakened, at the cardiac apex there is a soft systolic temperature, pain in the gastrocnemius muscles, icteric
murmur. What factors would be the most indicative of face, and dark urine. Objectively: body temperature –
the likely disease etiology? 38°C, blood pressure – 100/70 mm Hg, conjunctival
1. Creatine kinase haemorrhages, hepatosplenomegaly, and oliguria. What
2. Rheumatoid factor is the most likely provisional diagnosis?
3. Anti streptolysin O 1. Trichinosis
4. C reactive protein 2. Pseudotuberculosis
5. Seromucoid 3. Leptospirosis
4. Viral hepatitis
65. A 40-year-old victim of a traffic accident sustained 5. Brucellosis
the following injuries: closed diaphyseal femur fracture,
brain concussion, multiple rib fractures, 69. In the process of hiring, a prospective employee has
hemopneumothorax, degloving shin injuries. What undergone preventive medical examination and was
injuries require the most urgent attention? declared fit to work in this manufacturing environment.
1. Closed diaphyseal femur fracture What type of preventive medical examination was it?
2. All injuries are equivalent 1. Periodical
3. Degloving shin injuries 2. Specific
4. Multiple rib fractures, hemopneumothorax 3. Comprehensive
5. Brain concussion 4. Preliminary
5. Scheduled
66. To assess the effectiveness of medical technologies
and determine the power and direction of their effect on 70. A 56 year old woman was diagnosed with stage 2
the public health indicators, the research was conducted hypertension of the 2nd She belong to the group of
to study the immunization rate of children and measles moderate risk and has bronchial asthma. What group of
incidence rate by district. What method of statistical drugs is CONTRAINDICATED to this patient ?
analysis should be applied in this case? 1. Diuretics
1. Calculation of statistical significance of the difference 2. Angiotensin converting enzyme inhibitors
between two estimates 3. Calcium antagonist
2. Calculation of correlation coefficient 4. Imidazoline receptor antagonists
3. Calculation of morbidity index among the 5. B blockers
nonvaccinated
4. Calculation of standardized ratio 71. A 42-year-old man, a worker at the meat processing
5. Calculation of matching factor factory, developed an itching spot on his lower jaw,
which gradually transformed into a slightly painful
Krok 2- 2019 (General Medicine) (t.me/krok2GeneralMedicine) - 10

carbuncle 3 cm in diameter, surrounded by a painless 75. A 34 year old man on the 3rd day of ceftriaxone
swelling that reaches the clavicle. Temperature is treatment for acute otitis (daily dosage – 2 grams)
subfebrile, under 37.8 o The doctor suspects anthrax. developed Diarrhea occurring 5-6 times per day. Feces
What drug should this man be prescribed for treatment? are without mucus or blood admixtures. Temperature is
1. Interferon alpha 36.6 o Gregersen reaction (Occult blood in feces) is
2. Azidothymidin (Zidovudine) negative. Stool culture detected no pathogenic germs.
3. Penicillin What is the most likely cause of Diarrhea in this case?
4. Levomycetin (Chloramphenicol) 1. Antibiotic associated diarrhea
5. Biseptol (Co-trimoxazole) 2. Ulcerative colitis
3. Bacterial overgrowth syndrome
72. A 28-year-old woman complaining of irregular 4. Crohn‟s disease (Regional enteritis)
menstruations and infertility came to gynecological clinic. 5. Intestinal dysbiosis
Menstruations occur since the age of 15, irregular, with
delays up to 2 months. On examination she presents 76. A young man has made an appointment with the
with marked hirsutism and excessive body weight. On dermatologist. He complains of a painful facial rash in
vaginal examination the uterus is reduced in size and the beard and moustache area. This condition has been
painless. The ovaries on the both sides are dense and persisting for several weeks already. After shaving, the
enlarged. Ultrasound shows microcystic changes in the patient‟s condition aggravates. The diagnosis of sycosis
ovaries, the ovaries are 5×4 cm and 4.5× 4 cm in size is made. What primary morphological elements can be
with dense ovarian capsule. Basal body temperature is observed in the rash in this case?
monophasic . What is the most likely diagnosis? 1. Phlyctenae, maculae
1. Ovarian carcinoma 2. Pustules, papulae
2. Krukenberg tumor 3. Nodes, nodules
3. Endometrioid cysts 4. Maculae, nodes
4. Polycystic ovary syndrome 5. Pustules, bumps
5. Bilateral adnexitis
77. A woman came to the doctor with complaints of
73. In April during the medical examination of various increased body temperature upto 37.8 oC and
population groups, 27% of individuals presented with low moderately sore throat for the last 3 days. Objectively;
working ability and rapid fatigability. The following mandibular lymph nodes are enlarged upto 3cm.
symptoms were observed in the affected individuals: Palatine tonsils are hypertrophied and covered with gray
Swollen friable gingiva that bleeds when pressed, coating that spreads to the uvula and anterior pillars of
hyperkeratosis follicularis not accompanied by skin the fauces. What is the most likely diagnosis?
dryness. These symptoms most likely result from the 1. Oropharyngeal diphtheria
following pathology: 2. Agranulocytosis
1. C-hypovitaminosis 3. Oropharyngeal candidiasis
2. A-hypovitaminosis 4. Infectious mononucleosis
3. Polyhypovitaminosis 5. Pseudomembranous (Vincent‟s) tonsillitis
4. B1-hypovitaminosis
5. Parodontosis 78. In the impatient gynecological unit within a year 6500
women underwent treatment. They spent there a total of
74. An 18-year-old patient always obeys others and 102000 bed-days. What indicator of gynecological unit
adapts his needs to the demands of the people on whom work can be calculated based on these data?
he depends. He excessively defers to their wishes and 1. Bed turnover rate
makes them responsible for his well being, cannot 2. Average length of inpatient stay
defend his interests and needs support from other 3. Planned bed occupancy rate per year
people. Such psychic profile has been formed in the 4. Average bed occupancy rate per year
childhood, remains unchanged, and hinders adaptation. 5. Number of beds by hospital department
What psychic disorder is observed in this patient?
1. Anxiety (avoidant) personality disorder 79. A 25-year-old woman complains of fatigue,
2. Anankastic personality disorder dizziness, hemorrhagic rashes on the skin. She has
3. Psychopathy-like state been presenting with these signs for a month. Blood test:
4. Dependent personality disorder erythrocytes – 1.0 x 1012/L, Hb – 37 g/L, color index –
5. Markedly accentuated personality 1.1, leukocytes – 1.2 x 109/L, platelets – 42 x 109/L.
Krok 2- 2019 (General Medicine) (t.me/krok2GeneralMedicine) - 11

What analysis would be the most advisable for diagnosis costovertebral angle tenderness, pain on palpation of
making in this case? kidneys and along the ureter on the right. Urine test:
1. Liver biopsy proteins, fresh erythrocytes, leukocytes. Make the
2. Coagulation studies provisional diagnosis:
3. US of the gastrointestinal tract 1. Acute renal failure
4. Sternal puncture (bone marrow biopsy) 2. Acute glomerulonephritis
5. Splenic biopsy 3. Acute pyelonephritis
4. Urolithiasis, renal colic
80. A chronic alcoholic was hospitalized into the 5. Polycystic kidney disease
therapeutic inpatient unit due to pneumonia. On the day
5 of his hospitalization he became disoriented in time 84. A 48-year-old woman complains of disturbed
and space, developed fear-inducing visual hallucinations menstrual cycle: her periods last 7-9 days and are
and motor agitation. Full body tremor and tremor of the excessively profuse throughout the last half-year. She
limbs are observed. X-ray and physical examination notes occasional hot flashes in her head, insomnia,
detect the signs of his convalescence from pneumonia. irritability, and headaches. Her skin is of normal color.
What tactics should be chosen regarding his patient? Blood pressure- 150/90 mm Hg, pulse – 90/min.,
1. Compulsory medical treatment for alcoholism rhythmic. The abdomen is soft and painless. Bimanual
2. Transfer into the inpatient narcology department examination shows no uterine enlargement, the
3. Continue the treatment in the therapeutic department appendages cannot be detected. The vaginal fornices
4. Discharge from the hospital are free. What is the most likely diagnosis?
5. Transfer into the neuroresuscitation department 1. Premenstrual syndrome
2. Stein- Leventhal syndrome(polycystic ovary
81. A 72 year old man diagnosed with ischemic heart syndrome)
disease presents with diffuse cardiosclerosis, permanent 3. Uterine myoma
tachysystolic artrial fibrillation, heart failure IIa, FC III. 4. Climacteric syndrome
Objective examination of the vital signs: blood pressure 5. Adrenogenital syndrome
is 135/80 mmHg, heart rate is 160/min, pulse is 125/min.
Left ventricular ejection fraction is 32%, What drug is 85. A 58 year old woman came to the gynecological
indicated in this case and should be prescribed to the clinic. She complains of bloody discharge from her
patient? genital tracts. Menopause is 8 years. Gynecological
1. Ivabradine examination: the uterus is slightly enlarged, dense to
2. Digoxin touch, with limited mobility; the uterine appendages
3. Isadrine (Isoprenaline) cannot be detected; parametrium is free. Fractional
4. Procainamide (Novocainamide) curettage of the uterine cavity yields a significant amount
5. Verapamil of medullary substance in the scrape. What is the most
likely diagnosis?
82. The 5-year-old child has been ill for 2 weeks. Cough 1. Adenomyosis
attacks developed first and were then followed by 2. Uterine corpus cancer
reprises. During coughing the child‟s face turns red and 3. Hormone producing ovarian tumor
cervical veins bulge. The cough attacks induce vomiting, 4. Uterine cervix cancer
X-ray shows intensified bronchial pattern. Blood test: 5. Chorioepithelioma
leukocytes – 16 x 109/L, lymphocytes – 72%,
erythrocyte sedimentation rate – 4mm/hour. What is the 86. A newborn has Apgar score of 9. When should this
most likely diagnosis? infant be put to the breast?
1. Adenovirus infection 1. After 12 hours
2. Obstructive bronchitis 2. After 2 hours
3. Foreign body 3. In the delivery room
4. Pneumonia 4. On the 2nd day
5. Pertussis 5. On the 3rd day

83. A 45 year old man was brought by an ambulance 87. A 13-year-old girl for a month has been complaining
into the emergency hospital. He complains of sudden of fatigability, dull pain in her right subcostal area,
pain in the lumbar area, frequent painful urination and abdominal distension and constipation. Abdominal
vomiting. Examination detects pain in the lumbar area, palpation reveals positive Kehr, Murphy and Ortner
Krok 2- 2019 (General Medicine) (t.me/krok2GeneralMedicine) - 12

signs, while Desjasrdins and Mayo-Robson points are 1. Raynaud disease


painless. Total bilirubin is 14.7 mcmol/L, predominantly 2. Leriche Syndrome (Aortoiliac occlusive disease)
indirect, ALT – 20 U/L, AST – 40 U/L, amylase – 6.3 3. Obliterating endarteritis
mmol/L. Echocholecytography shows practically no 4. Deep thrombophlebitis
contraction of gallbladder. Make the provisional 5. Diabetic angiopathy
diagnosis?
1. Chronic pancreatitis 91. A 30-year-old woman made an appointment with the
2. Chronic hepatitis family doctor for scheduled vaccination of her 2-year-old
3. Acute pancreatitis child. What type of healthcare provide such medical
4. Hyperkinetic biliary dyskinesia services?
5. Hypokinetic biliary dyskinesia 1. Tertiary healthcare
2. Emergency aid
88. A 45 year old woman is registered for regular check 3. Primary healthcare
ups due to Werlhof disease (immune thrombocytopenia). 4. Palliative care
Complete blood count: Hb – 100 g/L, erythrocytes – 2.8 5. Secondary healthcare
x 1012/L, platelets – 90.0 x 109/L, leukocytes – 8.4 x
109/L, erythrocyte sedimentation rate – 13 mm/hour. 92. A 35-year-old patient developed an epileptic attack
Examination detects a single small hematoma on the with tonoclonic spasms that lasted for 3 minutes. After
anterior surface of the thigh, developed after the patient the attack the patient fell asleep but in 5 minutes the
accidently stumbled on a table. What treatment tactics second attack occurred. The first step of emergency aid
should be chosen in this case? would be to:
1. Continue the supervision by the hospital 1. Ensure patency of airways
haematologist 2. Introduce diazepam intravenously
2. Administer thrombolytic mass, continue with 3. Administer chloral hydrate via an enema
treatment in the haematology unit 4. Take blood from the vein for analysis
3. Urgent hospitalization into the haematology unit 5. Prescribe antiepileptic drugs
4. Urgent hospitalization into the general care unit
5. Urgently start a hemostatic therapy, followed by a 93. A 3-week-old infant developed large, flaccid vesicles
planned hospitalization into the haematology unit. with purulent contents on the skin of chest and
abdomen. The vesicles rupture quickly. Make the
89. A 7-year-old boy has severe pulmonary provisional diagnosis:
mucoviscidosis (cystic fibrosis). He complains of 1. Vesiculopustulosis
dyspnea and blood expectoration. Objectively he 2. Pemphigus neonatorum
presents with lagging physical development, 3. Pseudofurunculosis
acrocyanosis, hepatomegaly, drumstick fingers, and nail 4. Toxic erythema
plates resembling a “clock face”. Provisional diagnosis of 5. Pemphigus syphiliticus
chronic examination would be the most informative for
diagnosis confirmation? 94. In a rural health care area there is an increasing
1. Doppler echocardiography cervical cancer morbidity observed. The decision is
2. Electrocardiography made to conduct a medical examination of the women
3. Rheography of the pulmonary artery living in this locality. What type of medical examination is
4. Ultrasound of the liver it?
5. Chest X-ray 1. Target
2. Prelimiary
90. A 32-year-old man complains of pain in his legs that 3. Screening
intensifies during walking, intermittent claudication, 4. Regular
numbness of his toes, extremity coldness and inability to 5. Complex
walk more than 100 meters. When he sleeps, his leg
usually hangs down. The patient has been smoking 95. Clinical statistical investigation was performed to
since he was 16. He drinks alcohol in excess. The left determine effectiveness of a new pharmacological
leg is colder than the right one; the skin of the preparation for patients with ischemic heart disease.
extremities is dry. No pulse can be detected on the pedal What parametric test (coefficient) can be used to
arteries, while pulsation on the femoral arteries is estimate the reliability of the results?
retained. What is the most likely diagnosis? 1. Kolmogorov-Smirnov test
2. Wilcoxon signed rank test
Krok 2- 2019 (General Medicine) (t.me/krok2GeneralMedicine) - 13

3. Student‟s t-distribution 3. Tricuspid insufficiency


4. Sign test 4. Gastric tumor
5. Matching factor 5. Asteriovenous aneurysm

96. A 45-year-old man developed constricting 100. A 3-month-old child with signs of rickets presents
retrosternal pain that occurs during walks at the distance with positive Chvostek, Trousseau and Maslov signs.
of 200m. Objectively: heart rate is 80/min, BP is 160/90 One day ago the parents witnessed a cyanotic attack in
mmHg. During cardiopulmonary exercise test at 50 W their child – the child broke into a cold sweat, the eyes
there is a depression of S-T segment by 3 mm below the bulged and respiratory arrest occurred. One minute later
isoline in V3-V4. What is the provisional diagnosis? the child drew in a loud breath and child‟s condition
1. Alcoholic myocardiodystrophy normalized again. What is the cause of the described
2. Exertional angina pectoris, functional class II signs of the disease?
3. Somatoform autonomic dysfunction, hypertension 1. Decrease of blood phosphorus levels
type 2. Decrease of blood calcium levels
4. Exertional angina pectoris, functional class IV 3. Increase of blood calcium levels
5. Exertional angina pectoris, functional class III 4. Metabolic acidosis
5. Increase of blood phosphorus levels
97. A patient has gradually lost consciousness. The skin
is pale and dry. There is a smell of ammonia from the 101. The burns unit received a patient, who 6 hours ago
mouth. Respirations are deep and noisy. Heart sounds during a fire received flame burns. On the patient‟s body
are muffled, pericardial friction rub is present. Blood there is gray-brown area of necrosis that covers ¾ of the
pressure is 180/130 mmHg. Blood test: Hb – 80 g/L, body perimeter. Occasionally there are small blisters
leukocytes – 12 x 109/L, blood glucose – 6.4 mmol/L, with haemorrhagic contents and patches of shredded
urea – 50 mmol/L, creatinine – 1200 mcmol/L, blood epidermis. What local therapy is necessary in this case?
osmolarity – 350 mOsmol/L. No urinary excretion. Make 1. Necrectomy with xenotransplantation
the diagnosis: 2. Decompression necrectomy
1. Acute renal failure 3. Necrectomy with dermal autograft
2. Hyperosmolar coma 4. Blister puncture
3. Uremic coma 5. Chemical necrolysis
4. Acute disturbance of cerebral circulation
5. Hyperglycemic coma 102. At night a 2-year-old child with upper respiratory
tract infection suddenly developed dyspnea with labored
98. A 38-year-old patient has been brought by an inspiration. Objectively the skin is pale, perioral cyanosis
ambulance to the surgical department with complaints of and slight acrocyanosis are observed. Breathing is loud,
general weakness, indisposition, black stool. On respiration rate is 32/min. Jugular, supra and
examination the patient is pale, there are dotted infraclavicular fossae retract during breathing.
haemorrhages on the skin of his torso and extremities. Respiration is coarse on auscultation. Heart sounds are
On digital investigation there are black feces on the clear and sonorous, heart rate is 120/min. What
glove. Blood test: Hb – 108 g/L, thrombocytopenia. condition was complicated by the development of the
Anamnesis states that a similar condition was observed upper respiratory tract infection?
1 year ago. Make the diagnosis: 1. Obstructive bronchitis
1. Non specific ulcerative colitis 2. Bornchiolitis
2. Rectal tumor 3. Airway foreign body
3. Thrombocytopenia purpura 4. Bronchial asthma
4. Ulcerative bleeding 5. Stenosing laryngotracheitis
5. Hemophilia
103. A 24-year-old woman, a kindergarten teacher, has
99. A 65 year old woman on abdominal palpation been sick for 2 days already. Disease onset was acute.
presents with a tumor in the umbilical region and above She presents with elevated body temperature upto
it; the tumor is 13×8 in size, moderately painful, non- 38.0°C , pain attacks in her lower left abdomen, liquid
mobile, pulsing. On auscultation systolic murmur can be stool in small amounts with blood and mucus admixtures
observed. What is the most likely diagnosis? 10 times a day. Pulse – 98/min., blood pressure –
1. Bicuspid insufficiency 110/70mm Hg. Her tongue is moist and coated with
2. Abdominal aortic aneurysm white deposits. The abdomen is soft, the sigmoid colon
Krok 2- 2019 (General Medicine) (t.me/krok2GeneralMedicine) - 14

is painful and spastic. Make the provisional diagnosis: triangle. Aure-Rozanov and Gabay signs are positive.
1. Yersiniosis Make the provisional diagnosis:
2. Rotavirus infection 1. Acute appendicitis
3. Salmonellosis 2. Right sided renal colic
4. Shigellosis 3. Intestinal obstruction
5. Escherichiosis 4. Acute cholecystitis
5. Cecal tumor
104. In the factory cafeteria there was an outbreak of
food poisoning. Clinical presentation indicates 108. A 23-year-old man has accidentally swallowed
staphylococcal etiology of this disease. 15 people are brake fluid. After that he has been presenting with anuria
sick. To confirm the diagnosis of food poisoning, for 5 days already; his creatinine levels elevated up to
samples need to be sent to the laboratory. What 0.569mmol/L . What treatment tactics should be chosen
samples should be obtained for analysis? in this case?
1. Vomit masses 1. Detoxication therapy
2. Blood for hemoculture 2. Hemodialysis
3. Saliva 3. Antidotal therapy
4. Urine 4. Diuretics
5. Blood 5. Plasmapheresis

105. A 38-year-old woman developed a medical 109. A 2 years old child with persisting cough and sub
condition 7 days after her return from Bangladesh. febrile body temperature after a case of URTI developed
Periodical elevation of temperature was accompanied by dyspnea, cyanosis of the nasolabial triangle, percussion
chills and excessive sweating. She was diagnosed with dullness and weakened respiration in the lower lobe of
tropical malaria. Next day her condition further the right lung and a slight mediastinal displacement to
deteriorated: body temperature – 38 oC, inertness, the left. What pulmonary pathology is likely to cause this
periodical loss of consciousness, generalized seizures, clinical presentation?
tachycardia, hypotension and icteric skin. What 1. Emphysema
complication can be suspected in this case? 2. Pneumonia
1. Purulent meningitis 3. Pleurisy
2. Serious meningitis 4. Bronchitis
3. Acute hepatic failure 5. Atelectasis
4. Acute heart failure
5. Cerebral coma 110. A woman undergoing in patient treatment for viral
hepatitis type B developed headache, nausea, recurrent
106. A 55-year-old woman complaints of pain and vomiting, memory lapses, flapping tremor of her hands
popping sounds in her left knee joint, which occur when and rapid pulse. Sweet smell from her mouth is
she climbs the stairs. Occasionally during movements detected. Body temperature is 37.6 oC, heart rate is
her joint becomes “stuck”, 5 years ago she suffered a 89/min. What complication developed in the patient?
trauma of her left knee. Complete blood count and 1. Acute liver failure
biochemical blood analysis show normal results. X- ray 2. Hypoglycemic shock
shows marked osteosclerosis and osteophytes. The joint 3. Ischemic stroke
space is narrowed. Make the provisional diagnosis: 4. Meningoencephalitis
1. Osteoarthritis 5. Gastrointestinal haemorrhage
2. Rheumatoid arthritis
3. Reactive arthritis 111. At night a 63-year-old woman suddenly developed
4. Gouty arthritis an asphyxia attack. She has a 15 year long history of
5. Psoriatic arthritis essential hypertension and has a myocardial infarction 2
year ago. Objectively her position in bed is orthopneic,
107. A 25-year-old man was hospitalized with complaints the skin is pale, the patient is covered with cold sweat,
of pain in his lower abdomen and right lumbar area that acrocyanosis is observed. Pulse – 10/min. blood
appeared one hour ago. Patient‟s general state is pressure – 210/130 mmHg, respiration rate – 38/min.
moderately severe. Body temperature – 38.2 oC, heart Pulmonary percussion sound is clear, with slight
rate – 102/min. Tongue is dry. The abdomen is painful dullness in the lower segments; throughout the lungs
on deep palpation in the right iliac area and in the petit single dry crackles can be heard that become bubbling
Krok 2- 2019 (General Medicine) (t.me/krok2GeneralMedicine) - 15

and non-resonant in the lower segments. What is the diagnosis:


most likely complication in this patient? 1. Echinococcus
1. Acute left ventricular failure 2. Arnold-Chiari malformation
2. Paroxysmal tachycardia 3. Hydrocephalus
3. Pulmonary embolism 4. Hemorrhage
4. Acute right ventricular failure 5. Cerebral abscess
5. Bronchial asthma attack
115. Having studied the relationship between the
112. A 20-year-old man was hospitalized on the 9th day distance from villages to the local outpatient clinics and
of the disease. He attributes his disease to eating of frequency of visits to the clinics among the rural
insufficiently thermally processed pork. At its onset this population of this area, it was determined that the rank
condition manifested as periorbital edemas and fever. correlation coefficient in this case equals -0.9. how can
Objectively his body temperature is 38.5°C. The face is this relationship be characterized?
puffy and the eyelids are markedly swollen. Palpation of 1. Moderate inverse relationship
gastrocnemius muscles is sharply painful. Blood test 2. Strong direct relationship
shows hypereosinophilia. What is the etiology of this 3. Moderate direct relationship
disease? 4. Strong inverse relationship
1. Trichinella
2. Ascarididae 116. A newborn with gestational age of 31 weeks
3. Leptospira presents with hypotonia and depressed consciousness.
4. Trichuris Hematocrit is 35%, general cerebrospinal fluid analysis
5. Echinococci shows increased content of erythrocytes and protein,
and low glucose. These data correspond with clinical
113. For three years a 31-year-old woman has been presentation of:
complaining of pain and swelling of her radiocarpal and 1. Intracranial haemorrhage
metacarpophalangeal articulations and their reduced 2. Meningitis
mobility in the morning, which persisted upto 1.5 hours. 3. Sepsis
Two weeks ago she developed pain, swelling and 4. Intrauterine infection
reddening of her knee joints, her body temperature 5. Anemia
increased upto 37.5 o The treatment was untimely.
Examination of the internal organs revealed no 117. A 23-year-old man complains of facial edemas,
pathologic alterations. Diagnosis of rheumatoid arthritis headache, dizziness, low urinary output and urine
was made. What changes are most likely to be visible on discoloration (dark red). These complaints arose after a
the arthrogram? case of acute tonsillitis. On examination there are facial
1. Joint space narrowing, usuration edemas, the skin is pale, temperature is 37.4 oC, heart
2. Cyst in the subchondral bone rate is 86/min, blood pressure is 170/100 mmHg. Heart
3. Joint space narrowing, subchondral osteosclerosis sounds are muffled, the II heart sound is accentuated
4. Numerous marginal osteophytes over the aorta. What etiological factor is the most likely
5. Epiphyseal osteolysis in this case?
1. Staphylococcus saprophyticus
114. A 30-year-old man was brought to the 2. Streptococcus viridans
neurosurgical department with complaints of constant 3. Streptococcus pyogenes
headaches, nausea, vomiting, fever and weakness of 4. Beta hemolytic streptococcus
the right-side limbs. Anamnesis states that one month 5. Staphylococcus aureus
ago the patient had a surgery for left-sided suppurative
otitis and mastoiditis. He has been undergoing treatment 118. A newborn girl has apgar score of 7-8 points at the
in an ENT department. Approximately 2 weeks ago the 1-5 minutes after birth. During the labor there was a brief
temperature increased, and the patient developed difficulty with extraction of the shoulder girdle. After birth
headaches. Objectively: heart rate – 98/min., BP – the baby presents with disturbed function of the proximal
140/90 mm Hg, temperature- 38.3°C. neurologically segment and forced position of the right arm. The
manifested stiff neck: bilateral Kerning‟s symptom, shoulder is rotated inwards, the elbow is extended, the
unsteadiness during Romberg‟s maneuver. Computer forearm is pronated and the whole upper limb resembles
tomography of the brain revealed a three- dimensional an arm of a doll. What is the most likely clinical diagnosis
growth with a capsule in the left hemisphere. Make the in this case?
Krok 2- 2019 (General Medicine) (t.me/krok2GeneralMedicine) - 16

1. Intracranial haemorrhage 3. Cystitis


2. Erb-Duchenne palsy 4. Glomerulonephritis
3. Soft tissue injury of the right arm 5. Vulvitis
4. Thoracic spine trauma
5. Osteomyelitis of the right arm 123. After a long drive with the window open a man
developed facial asymmetry. He cannot close his right
119. 3 hours after a trauma, a young man developed eye, his right nasolabial fold is smoothed out,
bradycardia of 46/min, anisocoria D>S, hemi movements of expression are absent on the right, there
hyperreflexia S>D, hemiphypesthesia on the left, and a is a disturbance of gustatory sensation in the tongue on
convulsive disorder. The character of this process needs the right. No other neurological pathologies were
to be clarified. What method of examination will be most detected. What disease can be provisionally diagnosed
accurate for this purpose? in this patient?
1. Lumbar puncture 1. Trigeminal ganglionitis
2. Skull X-ray 2. Ischemic stroke
3. Echoencephalography 3. Neuropathy of the facial nerve
4. Electroencephalography 4. Neuropathy of the trigeminal nerve
5. Brain CT 5. Neuropathy of the oculomotor nerve

120. During an outdoors school event in hot weather, a 124. The body of a 24-year-old woman with suspected
10 year old girl lost her consciousness. Body poisoning has been found on the street. Forensic
temperature – 36.7 o Objectively her skin is pale and medical examination was requested on the site and the
cold to touch, her pupils are dilated. Blood pressure – body. According to the Criminal Procedure Code
90/50 mmHg. Heart rate – 58/min. What pathology, currently in force in Ukraine, forensic medical
occurred in this case? examination is required when it is necessary to
1. Syncope determine the:
2. Sympathicotonic collapse 1. Cause of death
3. Sunstroke 2. Mechanism of death
4. Paralytic collapse 3. Manner of death
4. Time of death
121. An 18-year-old girl was brought into the 5. Mode of death
gynaecology department with complaints of elevated
body temperature up to 37.8°C, sharp pain in her lower 125. During medical examination of high and middle
abdomen, more intense on the right and difficult school students, the doctors were assessing correlation
defecation. Vaginal examination detected a painful between biological and calendar age of the school
dense elastic formation 5×6 cm in the area of her right students based on the following criteria; height growth
ovary. Pregnancy test is negative. What is the most rate per year, ossification of the carpal bones, the
likely diagnosis? number of permanent teeth. What additional
1. Ovarian cyst rupture development criterion should be assessed at this age?
2. Appendicitis 1. Chest circumference
3. Ovarian apoplexy 2. Development of secondary sex characteristics
4. Torsion of ovarian tumor pedicle 3. Body mass
5. Ectopic pregnancy 4. Hand strength
5. Vital capacity of lungs
122. A 8 year old girl complains of frequent painful
urination in small amounts and urinary incontinence. The 126. A 27 year old woman complains of foul smelling
signs have been present for 2 days already. She discharge from her genital tracts, pain in her lower
complains her disease by overexposure to cold. abdomen and elevated temperature. The complaints
Costovertebral angle tenderness is absent. Complete arose 2 days ago. She has a history of surgical abortion
blood count is without pathologies. Urine test: at the term of 8 weeks one week ago. Mirror
leukocytes: 20-30 in the vision field, erythrocytes: 40-50 examination: the uterine cervix is clear, external orifice
in the vision field, unchanged, bacteriuria. What is the produces foul smelling discharge. Vaginal examination:
most likely diagnosis? the uterus lies in anteflextion, is mobile, painful, and
1. Urolithiasis slightly enlarged. The appendages are without changes.
2. Pyelonephritis Make the provisional diagnosis:
Krok 2- 2019 (General Medicine) (t.me/krok2GeneralMedicine) - 17

1. Salpingoophoritis 131. At the railroad crossing a passenger train collided


2. Acute respiratory disease with a bus. In this collision 26 bus passengers died,
3. Enterocolitis another 18 passengers received mechanical injuries of
4. Postabortal endometritis varying severity. Where will be professional medical aid
5. Appendicitis provided for the victims of this accident. Who will provide
this aid?
127. A 16 year old patient has made an appointment 1. At the site of the accident; specialized second
with an otolaryngologist. He complains of elevated body response emergency teams
temperature and sore throat. Disease onset was 2 days 2. In medico prophylactic institutions; general physicians
ago, after the patient are two portions of ice cream. and surgeons
Pharygoscopy shows hyperemic mucosa of the palatine 3. At the site of the accident; first response emergency
tonsils with purulent exudate in the lucanae. Make the teams
provisional diagnosis: 4. In medical institutions; all listed type of healthcare
1. Follicular tonsillitis workers
2. Pseudomembranous (Vincent‟s) tonsillitis 5. In medico-prophylactic institutions; specialized
3. Lacunar tonsillitis second response emergency teams
4. Diphtheria
5. Acute pharyngitis 132. The dermatologist has an appointment with a 30-
year-old man that complains of severely itching rashes
128. A boy had a foreign body removed from under his that especially disturb him at night. The rashes
nail plate. 3 days later he developed a sharp throbbing developed 2 week ago, after he had returned from a
pain at the end of his distal phalanx, which intensifies travel. Objectively on the lateral surfaces of his fingers,
when the phalanx is pressed, hyperemia of the nail fold, hands, writs, elbows, lower abdomen, genitals and
elevated body temperature upto 38.5 oC, and nail plate thighs there are paired papulovesicles, single pustules,
discoloration. Make the diagnosis: and scratch marks. What disease can be suspected?
1. Erysipelas 1. Shingles
2. Abscess 2. Pyoderma
3. Sublingual panaritium 3. Dermatitis
4. Erysipeloid 4. Eczema
5. Paronychia 5. Scabies

129. Increased general morbidity of the local population 133. A 35 year old pregnant woman with degree 1
is observed in the area near a factory, where essential hypertension, developed edemas and
atmosphere is being intensively polluted with sulphurous headache at the 33 week of her pregnancy. Objectively
gas. What effect does polluted air have on human body her general condition is satisfactory, blood pressure –
in this case? 160/100 mmHg, normal uterine tone. Fetal heart rate is
1. Chronic specific 140/min and rhythmic. She was diagnosed with daily
2. Acute non specific proteinuria – 4 g/L, daily diuresis – 1100 mL. Creatinine
3. Chronic non specific – 80 mcmol/L, urea – 7 mmol/L, platelets – 100 x 109/L.
4. Acute specific What complication of pregnancy occurred?
5. Selective 1. Mild preeclampsia
2. Hypertensive crisis
130. A 46-year-old woman came to the maternity clinic 3. Moderate preeclampsia
with complaints of moderate blood discharge from the 4. Severe preeclampsia
vagina, which developed after the menstruation delay of 5. Renal failure
1.5 months. On vaginal examination; the cervix is clean;
the uterus is not enlarged, mobile, painless; appendages 134. In the air of the feed kitchen at the poultry factory,
without changes. Make the diagnosis: at the area where formula feed is being mixed, the dust
1. Dysfunctional uterine bleeding concentration reaches 200 mg/m3. Air microflora is
2. Submucous uterine myoma represented predominantly by Asperquillus and Mucor
3. Adenomyosis fungi. What effect determines pathogenic properties of
4. Cancer of uterine body the dust?
5. Ectopic pregnancy 1. Fibrogenic
2. Mutagenic
Krok 2- 2019 (General Medicine) (t.me/krok2GeneralMedicine) - 18

3. Allergenic days. Objectively: kidneys cannot be palpated,


4. Toxic suprapubic area is without alterations, external genitalia
5. Teratogenic are non-pathologic. On rectal investigation: prostate is
not enlarged, painless, has normal structure. Cystoscopy
135. A 13-year-old girl for the last two weeks has been revealed no changes. What is the most likely diagnosis?
complaining of dyspnea and shin and foot edemas that 1. Necrotic papillitis
appear after physical exertion. In the morning the 2. Bladder tuberculosis
edemas significantly decrease. Clinical examination 3. Dystopic kidney
revealed enlarged liver and coarse systolic murmur over 4. Varicocele
the heart area. Blood test and urinalysis are without 5. Renal carcinoma
changes. What is the most likely cause of edemas in this
child? 140. A pregnant woman is 28 years old. Anamnesis
1. Angioneurotic edema accelerated labor complicated by the II degree cervical
2. Heart failure rupture. The following two pregnancies resulted in
3. Hepatic cirrhosis spontaneous abortions at the terms of 12 and 14 weeks.
4. Acute pyelonephritis On mirror examination: the uterine cervix is scarred from
5. Nephrotic syndrome previous ruptures at 9 and 3 hours, the cervical canal is
gaping. On vaginal examination: the cervix is 2 cm long,
136. During examination a 4-month-old child with the external orifice is open 1 cm wide, the internal orifice
meningococcemia presents with acrocyanosis, cold is half open; the uterus is enlarged to the 12th week of
extremities, tachypnea and thread pulse, anuria and pregnancy, soft, mobile, painless, the appendages are
sopor. What clinical syndrome is it? without changes. What diagnosis can be made?
1. Toxic shock syndrome 1. Incipient abortion, habitual non carrying of pregnancy
2. Acute renal failure 2. Cervical hysteromyoma, habitual non-carrying of
3. Encephalic syndrome pregnancy
4. Neurotoxicosis 3. Isthmico-cervical insufficiency, habitual non carrying
5. Exicosis of pregnancy
4. Threatened spontaneous abortion
137. During medical examination a cadet in the naval 5. Cervical pregnancy, 12 weeks
college was detected to have a painless dense ulcer
1.5×0.5 in size in his perianal area at the 2 O‟clock 141. A 58-year-old man complains of weakness and
position. The ulcer floor resembles „old fat‟. What is the tumor like formations that appeared on the anterior
provisional diagnosis? surface of his neck and in the inguinal region. Palpation
1. Hard syphilitic chancre of the rectum detects soft painless mobile cervical and inguinal lymph
2. Anal cancer nodes up to 2 cm in diameter. The liver protrudes by 2
3. Anal crypt suppuration cm from the edge of the costal margin, the lower splenic
4. Rectal fissure pole is at the umbilical level. In blood: erythrocytes – 3.5
5. Rectal fistula x 1012/L, Hb – 88 g/L, leukocytes – 86 x 109/L, band
neutrophils – 1%, segmented neutrophils – 10%,
138. A 1.5-month-old child on breastfeeding presents lymphocytes – 85%, eosinophils – 2%, basocytes – 0%,
from birth with daily vomiting, irregular liquid foamy feces monocytes – 2%, erythrocyte sedimentation rate – 15
and meteorism, which are resistant to antibacterial and mm/hour, Gumprecht shadows. What is the most likely
probiotic therapy; no increase of body mass is observed. diagnosis?
The child‟s condition improved, when breastmilk was 1. Chronic myeloleukemia
substituted with „NAN low lactose‟ formula. What 2. Acute leukemia
pathology is it? 3. Chronic lymphatic leukemia
1. Infectious enteritis 4. Lymphogranulomatosis
2. Lactase deficiency 5. Lymphocytic leukemoid reaction
3. Intestinal lambliasis (Giardiasis)
4. Drug induced enteritis 142. A 39-year-old man suffers from chronic adrenal
5. Functional dyspepsia insufficiency and receives replacement glucocorticoid
therapy (hydrocortisone – 15 mg/day). He is to undergo
139. A 50-year-old patient was brought to a hospital with elective surgery for calculous cholecystitis. What
complaints of blood in urine. Urination is painless and medication adjustment should be made on the day of the
undisturbed. Macrohematuria had been observed for 3
Krok 2- 2019 (General Medicine) (t.me/krok2GeneralMedicine) - 19

surgery to prevent the development of acute adrenal 146. A 25-year-old woman has been suffering form
insufficiency? diabetes mellitus since she was 9. She was admitted
1. Prescribe a large volume intravenous fluid infusion into a nephology unit with significant edemas of the face,
2. Add a mineralocorticoid arms and legs. BP – 200/110 mmHg, Hb – 90 g/L, blood
3. Add an antibiotic creatinine – 850 mcmol/L, urine proteins – 1.0 g/L,
4. Increase the dosage by 2-3 times leukocytes – 10-15 in the vision field. Glomerular
5. Cancel the drug for the day of the surgery filtration rate – 10 mL/min. What tactics should the
doctor choose?
143. A 65-year-old man complains of dyspnea, severe 1. Transfer into the endocrinology clinic
cough with expectoration of small amount of blood 2. Transfer into the hemodialysis unit
streaked sputum, weight loss, body temperature 37.2 3. Dietotherapy
oC, loss of appetite and weakness. He has been 4. Renal transplantation
suffering from this condition for years. The patient‟s 5. Active conservative therapy for diabetic nephropathy
condition deteriorated one year ago, dyspnea developed
3 weeks ago. The patient is a lifelong smoker. He is a 147. On the day 4 after the caesarean section a woman
carpenter by occupation. Objectively he is of normal developed fever with body temperature upto 39oC and
body type but emaciated. Right side of the chest is abdominal pain. Pulse – 104/min. She vomited twice.
retracted, excursions are limited, accessory muscles The patient is sluggish, her tongue is dry and has gray
take part in the respiration, respiratory rate is 22/min. coating. The abdomen is distended. Signs of peritoneal
Percussion detects dull sound over the right upper irritation are positive in all segments. Peristalsis cannot
segment. Chest X-ray shows shrunken right upper lobe be auscultated. No passage of gas occurs. Uterine
with homogeneous shadow connected to the root of the fundus is located at the level of the navel. The uterus is
lung; the root is deformed; mediastinal organs are painful on palpation. The discharge is moderate and
displaced to the right. What is the most likely diagnosis? contains blood and pus. What is the most likely
1. Pulmonary sarcoidosis diagnosis?
2. Pulmonary tuberculosis 1. Diffuse peritonitis
3. Complete right-sided pneumothorax 2. Progressive thrombophlebitis
4. Fibrosing alveolitis 3. Metroendometritis
5. Obstructive atelectasis 4. Pelvic peritonitis
5. Parametritis
144. A 38-year-old woman complains of weakness,
sleepiness, pain in the joints, weight gain despite low 148. Having examined a 52 year old patient, the doctor
appetite, and constipations. She presents with dry and diagnosed him with obesity (body mass index- 34 kg/m2,
thickened skin, puffy and amimic face, narrowed waist circumference – 112 cm) and arterial hypertension
palpebral fissures, thick tongue and deep hoarse voice. ( 170/105 mm Hg). 2- hour postprandial blood sugar is
Her heart sounds are weak, pulse is 56/min. Low levels 10.8 mmol/L. What biochemical blood analysis needs to
of free T4 are observed. This patient needs to take the be conducted to diagnosis the patient with metabolic
following on a regular basis: syndrome X?
1. Furosemide 1. Lipid profile
2. Calcium gluconate 2. Calcium and phosphorus
3. Thyroxine 3. Electrolytes
4. Lithium carbonate 4. Bilirubin
5. Mercazolil (Thiamazole) 5. Creatinine and urea

145. On laboratory investigation of a pork sample there 149. A 15 year old girl complains of dizziness and
is 1 dead trichinella detected in 24 sections. This meat sensation of lack of air that she develops in emotionally
should be: straining situations. Relief occurs after she takes
1. Frozen until the temperature of -10 oC is reached in corvalol. Objectivrly: hyperhidrosis and marble-like
the deep layers, with subsequent exposure to cold for 15 pattern of the skin of her palms and feet. Clinical and
days. instrumental examination revealed no organic changes
2. Processed for boiled sausage production in the central nervous, cardiovascular, and respiratory
3. Sent for technical disposal systems. What provisional diagnosis can be made?
4. Allowed for sale with no restriction 1. Somatoform autonomic dysfunction
5. Processed and sold through public catering network 2. Stenosing laryngotracheitis
Krok 2- 2019 (General Medicine) (t.me/krok2GeneralMedicine) - 20

3. Bronchial asthma foods, bitter taste in her mouth in the morning,


4. Acute epiglottitis constipations, and flatulence. Objectively she has
5. Obstructive bronchitis excess weight, her body temperature is 36.9°C; there is
a coating on the root of her tongue; the abdomen is
150. Indicators of work of impatient departments in the moderately distended and painful in the area of
city hospitals for the past year were analysed. After that gallbladder projection. What examination would be the
the meeting was held at the central city hospital and most helpful for diagnosis- making?
based on the results of the analysis, a decision was 1. Ultrasound
made to decrease the number of beds in the impatient 2. Liver scanning
departments and instead open daycare units with partial 3. Duodenal intubation
hospitalization in the city polyclinics. What is the main 4. Cholecystography
goal of this decision? 5. Duodenoscopy
1. Decreased mortality during impatient treatment
2. Optimization of the average bed occupancy rate per 154. A 37 year old man suddenly developed acute
year headache accompanied by nausea, vomiting, and
3. Rational use of the bed fund impaired consciousness. Objectively blood pressure is
4. Intensification of the bed fund usage 190/120 mmHg, the face is hyperemic. Patient‟s
5. Decrease of the length of impatient stay consciousness is clouded, his answers to the questions
are short, monosyllabic. Movement and sensory
151. Mother of a 5-year-old child noticed on the head of disturbances are absent. Meningeal signs are positive.
her child a round „bald‟ spot 3 cm in diameter. All the Cerebrospinal fluid contains blood. What provisional
hairs in the focus are broken off at the lengths of 5-6 diagnosis can be made?
mm. The day before the child was petting a stray cat. 1. Meningitis
Make the diagnosis: 2. Ischemic stroke
1. Alopecia areata 3. Encephalitis
2. Microsporia 4. Subarachnoid haemorrhage
3. Psoriasis 5. Cerebral vascular embolism
4. Superficial trichophytosis
5. Deep trichophytosis 155. 10 hours after birth a child developed jaundice,
hypotonia, hyporeflexia and moderate
152. A 19-year-old young man complains of cough with hepatosplenomegaly. Feces and urine are of normal
expectoration of purulent sputum in the amount of 100 color. Umbilical cord blood bilirubin is 51 mcmol/L due to
mL per day, haemoptysis, dyspnea, increased body unconjugated bilirubin levels. In venous blood:
temperature up to 37.8°C, general weakness, weight erythrocytes – 3.5 x 1012/L, Hb – 140 g/L, reticulocytes
loss. The patient‟s condition lasts for 4 years. – 1.5%, bilirubin – 111 mcmol/L, conjugated – 11
Exacerbations occur 2-3 times a year. The patient mcmol/L, ALT – 40 U/L, AST – 30 U/L. mother‟s blood
presents with malnutrition, pale skin, cyanosis of the lips, group is A(II) Rh(-), child‟s blood group is A(II) Rh(+).
drumstick (clubbed) fingers. Tympanic percussion sound What laboratory test can confirm the diagnosis?
in the lungs, weakened repiration, numerous various 1. Measurement of glucose 6 phosphate
moist crackles in the lower pulmonary segments on the dehydrogenase levels in erythrocytes
left can be observed. In blood: erythrocytes – 3.2 x 2. Erythrocytometry
1012/L, leukocytes – 8.4 x 109/L, ESR – 56 mm/hour. 3. Coombs test
On X-ray: lung fields are emphysematous, the left 4. Measurement of erythrocyte osmotic resistance
pulmonary root is deformed and dilated. What is the 5. Viral hepatitis markers analysis
most likely diagnosis?
1. Chronic left-sided pneumonia 156. A 35 year old woman complains of high body
2. Multiple bronchiectasis of the left lung temperature and pain in the upper outer quadrant of her
3. Suppuration of the cyst in the left lung right buttock, which developed after an injection. She
4. Left-sided pulmonary cystic dysplasia has been presenting with this condition for 3 days. At the
5. Chromic abscess of the left lung site of injection the skin is hyperemic; there is painful
infiltrate with an area of softening in its center. The
153. A 52-year-old woman has been suffering for 2 woman is diagnosed with a post injection abscess of the
years from dull, occasionally exacerbating pain in her right buttock. What tactics should the surgeon choose in
right subcostal area, occurring after eating high-fat this case
Krok 2- 2019 (General Medicine) (t.me/krok2GeneralMedicine) - 21

1. Hospitalization, prescription of antibiotics, UHF 160. A 16 year old girl has primary amenorrhea, no
2. Abscess incision, sanation and drainage of the cavity pubic hair growth, normally developed mammary glands;
3. Antipyretic agents, massage and application of dry her genotype is 46 XY; uterus and vagina are absent.
heat to the right buttock What is your diagnosis?
4. Abscess puncture, pus removal followed by 1. Sheehan syndrome
application of antiseptics 2. Cushing syndrome
5. 10-15 minutes of low intensity laser radiation directed 3. Cushing disease
at the right buttock 4. Testicular feminization syndrome
5. Mayer-Rokitansky-Kuster-hauser syndrome
157. Heart X-ray of a 31-year-old man has been
revealed the following: with tightly filled opacified 161. A 24- year-old pregnant woman on her 37th week
esophagus there is a marginal filling defect in its middle of pregnancy has been brought to the maternity obstetric
third on the posterior wall; the defects is 1.8×1.3 cm in service with complaints of weak fetal movements. Fetal
size with clear oval border. Mucosal folds are retained heartbeats are 95/min. on vaginal examination the
and envelop the defect; wall peristalsis and elasticity are uterine cervix is tilted backwards, 2cm long, external
not affected. There are no complaints regarding the orifice allows inserting a fingertip. Biophysical profile of
condition of the patient‟s alimentary canal. Make the the fetus equals 4 points. What tactics of pregnancy
provisional diagnosis: management should be chosen?
1. Esophageal tumor 1. Treatment of placental dysfunction and repeated
2. Achlasia cardiae analysis of the fetal biophysical profile on the next day
3. Diverticulum 2. Urgent delivery via a cesarean section
4. Barrett esophagus 3. Urgent preparation of the uterine cervix for delivery
5. Esophageal burns 4. Doppler measurement of blood velocity in the
umbilical artery
158. A 51-year-old man complains of vomiting with 5. Treatment of fetal distress; if ineffective, then elective
blood. He has been drinking alcohol excessively. Health cesarean section on the next day
disorder has been observed since he was 40, when he
first developed jaundice. On examination the skin and 162. A 45-year-old man with thrombophlebitis of the
visible mucosa are icteric, with a stellate vascular deep veins in this legs suddenly after physical exertion
pattern. The patient is malnourished and presents with developed sharp pain in his thorax on the right, dyspnea
abdominal distension, umbilical hernia and ascites. The and hemoptysis. Objectively his condition is severe; he
edge of the liver is tapered and painless, +3cm, the presents with acrocyanosis, shortening of pulmonary
spleen is +2cm. Blood test: Hb- 80g/L, leukocytes – 3 x percussion sound on the right and weakened respiration.
109/L, platelets – 85 x 109/L. What is the cause of portal Respiration is 30/min, blood pressure is 110/80 mmHg.
hypertension in this patient? ECg shows sinus tachycardia, heart rate is 120/min,
1. Constrictive pericarditis electrical axis of the heart deviates to the right, SI-QIII.
2. Hepatic cirrhosis What is the most likely diagnosis?
3. Budd-Chiari syndrome 1. Cancer of the right lung
4. Thrombosis of the splenic vein 2. Spontaneous pneumothorax
5. Hemochromatosis 3. Community acquired right sided pneumonia
4. Pulmonary embolism
159. A 13-year-old girl has 30% of excessive body mass, 5. Right sided exudative pleurisy
she started to gain weight at the age of 3. She has a
family history of obesity. Her height and sexual 163. A 10 year old boy was brought into the hospital with
development are normal for her age. The appetite is complaints of expiratory dyspnea, respiration are 30/min.
excessive. She complains of periodical headaches. He explains his state by a change in the weather
Blood pressure – 120/80 mmHg. Subcutaneous fat is conditions. For the last 4 years the body has been
evenly distributed, she has no stretch marks. There is registered for regular check-ups due to his diagnosis of
juvenile acne on her face. What type of obesity is it? third degree persistent bronchial asthma. To provide
1. Hypothalamic obesity emergency aid for this child, first he needs to be given:
2. Hypothalamic syndrome of puberty 1. Claritin (Loratadine)
3. Alimentary constitutive obesity 2. Dexamethasone
4. Hypothyroid obesity 3. Salbutamol or short acting β2 agonists
5. Adrenal obesity
Krok 2- 2019 (General Medicine) (t.me/krok2GeneralMedicine) - 22

4. Euphylline (Aminophylline) interscapular region and aggravates during a deep


5. Adrenaline intake of breath. Dyspnea occurs during physical
exertion. Vesicular respiration in the lungs is weakened.
164. A 23-year-old man complains of severe pain in his Heart sounds are rhythmic, heart rate is 86/min, blood
left knee joint. Objectively the left knee joint is enlarged, pressure is 135/80 mmHg. The abdomen is soft and
with hyperemic skin, painful on palpation. Complete painless. X-ray shows micronodular pulmonary fibrosis.
blood count: erythrocytes – 3.8 x 1012/L, Hb- 122g/L, Make the provisional diagnosis.
leukocytes – 7.4 x 109/L, platelets – 183 x 109/L. 1. Berylliosis
Erythrocyte sedimentation rate – 10 mm/hour. Bleeding 2. Siderosis
time (Duke method) – 4 min., Lee- White coagulation 3. Metal pneumoconiosis
time – 24 min. Partial thromboplastin time (activated) – 4. Byssinosis
89 seconds. Rheumatoid factor – negative. What is the 5. Carboconiosis
most likely diagnosis?
1. Hemorrhagic vasculitis (Henoch – Schonlein 168. A 22-year-old postparturient woman on the 12th
purpura), articular form day after normal child birth informs of elevated body
2. Hemophilia, hemarthrosis temperature up to 39° C for the last 3 days and pain in
3. Thrombocytopathy her right mammary gland. The right mammary gland is
4. Rheumatoid arthritis enlarged, hot to touch, tense, hyperemic, and painful.
5. Werlhof disease (immune thrombocytopenia) Palpation reveals there a dense infiltration 8×8 cm with a
fluctuation in its center. What is the most likely
165. It is the 3rd day after the first normal term labor; the diagnosis?
infant is rooming in with the mother and is on 1. Postpartum period, day 12. Right-sided infiltrative-
breastfeeding. Objectively; the mother‟s general purulent mastitis
condition is satisfactory. Temperature is 36.4 oC, heart 2. Postpartum period, day 12. Right-sided lactostasis.
rate is 80/min, BP is 120/80 mmHg. Mammary glands, 3. Postpartum period, day 12. Right-sided phlegmonous
are soft and painless; lactation is moderate, unrestricted mastitis
milk flow. The uterus is dense, the uterine fundus is 4. Postpartum period, day 12. Right-sided serous
located 3 finger widths below the navel. Lochia are mastitis.
sanguine-serous, moderate in volume. Assess the 5. Postpartum period, day 12. Right-sided gangrenous
dynamics of uterine involution: mastitis.
1. Pathologic involution
2. Physiological involution 169. A 48-year-old woman has arrived to the surgical
3. Lochiometra unit with wounds in her thigh. On examination the wound
4. Hematometra surface has dirty-gray coating with unpleasant sweet
5. Subinvolution smell. Wound content resembles raspberry jelly. Skin
tissues around the wound are glossy and turgid.
166. A 35-year-old man suffers from insulin-dependent Palpation reveals moderate crepitation in the tissues.
diabetes mellitus and chronic cholecystitis. He takes What microflora is the most likely to cause such
NPH insulin; 20 units in the morning and 12 units in the inflammation?
evening. After a meal he developed pain in the right 1. Streptococci
subcostal area, nausea, vomiting, sleepiness and 2. Anaerobic clostridial
increased polyuria. What prehospital measures will be 3. Anaerobic non-clostridial
the most effective for prevention of crisis within the next 4. Staphylococci
several hours? 5. Blue push bacillus
1. Decrease carbohydrates in the diet
2. Change insulin regimen 170. A 30-year-old man came to the family physician. 2
3. Take cholagogues months ago he underwent a surgery for open fracture of
4. Exclude fats from the diet the humerus. On examination the patient‟s condition is
5. Take analgesics satisfactory; in the area of postoperative wound there is
a fistula that discharges a small amount of pus; the area
167. A 43-year-old man, a coal face worker with 15 year itself is red; fluctuation is detected. X-ray shows
long record of work, complains of cough, thoracic pain destruction of the humerus with sequestra. What
and dyspnea. The cough is mild, usually dry, occurs complication did the patient develop during the
mostly in the morning. The pain is localized in the postoperative period?
Krok 2- 2019 (General Medicine) (t.me/krok2GeneralMedicine) - 23

1. Suture sinus 4. Immunogram


2. Post traumatic phlegmon 5. Micturating cystourethrography
3. Post traumatic osteomyelitis
4. Hematogenous osteomyelitis 175. A 73-year-old woman came to the family physician
5. Wound suppuration for one of her regular follow–up examinations. Three
months ago, she was found to have type 2 diabetes
171. A 57-year-old woman during a regular ultrasound mellitus. She was keeping to her diet and exercise plan
examination presented with a space-occupying and taking phytopreparations. On examination her
heterogeneous lesion in the right kidney. What is the fasting glucose was within the range of 7.8-8.6 mmol/L,
most informative method of renal tumor diagnostic? HbA1c – 7.9%. Height – 164 cm, weight – 83 kg. What
1. Three glass urine test blood sugar controlling medicine should she be
2. Excretory urography prescribed first in the course of her pharmacological
3. Spiral computed tomography therapy?
4. Radioisotope renography 1. Glibenclamide
5. Retrograde pyelography 2. Gliclazide
3. Glimepiride
172. A 10 year old girl exhibits high level of physical 4. Metformin
development, her body length increased by 10 cm within 5. Insulin
a year (which is double the norm of her age group), the
number of permanent teeth corresponds with the age 176. A 63-year-old man complains of unmotivated
norm (20), the development of her secondary sex weakness and pressing and bursting sensation in the left
characteristics is 3 years ahead of her age (Ma, P, Ax, subcostal area. According to him, these signs have been
menarche). Development rate ahead of her biological present for a year already. Previously he was healthy.
age can occur due to: He took part in containment measures during the
1. Deficient hygienic education accident at the Chornobyl Nuclear power plant.
2. Certain components of her diet Objectively the skin is pale, peripheral lymph nodes are
3. Acceleration not enlarged, the liver is +3cm, the spleen is +10cm .
4. Endocrine disorders Complete blood count: erythrocytes – 3.1 x 1012/L, Hb-
5. Sports training 100g/L, leukocytes – 46 x 109/L, blasts – 2%,
promyclocytes – 10%, myclocytes – 18%, band
173. A 46-year-old woman has Diarrhea with abdominal neutrophils – 27%, segmented neutrophils – 10%,
distension, loss of body mass, and large amounts of lymphocytes- 12%, eosinophils – 6%,basocytes – 3%,
porridge like foul smelling stool without blood steaks or monocytes – 2%, erythrocyte sedimentation rate – 20
tenesmus. Objectively examination detects moderate mm/hour. What is the most likely diagnosis?
tenderness in the mesogastrium and left abdominal 1. Acute leukemia
flank. Feces analysis detects steatorrhea with neutral fat 2. Chronic myeloleukemia
and creatorrhea. What prescription would be the most 3. Hemolytic anemia
advisable in this case? 4. Chronic lymphatic leukemia
1. Cholinergic antagonists 5. Hepatic cirrhosis
2. Multi enzyme preparations
3. Metronidazole and Loperamide 177. A 10 year old boy with symptoms of arthritis and
4. Cholinergic antagonists and antibacterial agents myocarditis was brought to a hospital. Based on clinical
5. Antacids and antispasmodics examination the provisional diagnosis of juvenile
rheumatoid arthritis was made. What symptom is the
174. A 7 year old girl has been twice treated with most contribute for the diagnostics of this disease?
antibacterial agents for urinary tract infection. US shows 1. Affection of the large joints
no severe renal defects. The child presents with 2. Enlarged heart
recurrence of leukocyturia and bacteriuria, elevated body 3. Regional hyperemia of the joints
temperature upto 38.5 oC, and pain in her left lumbar 4. Increased heart rate
area. What examination should be conducted first to 5. Reduced mobility of the joints in the morning
clarify the cause of urinary infection recurrence?
1. Excretory urography 178. A 39-year-old man, a battery attendant, suddenly
2. Retrograde pyelography developed weakness, loss of appetite, nonlocalized
3. Radioisotope renography colicky abdominal pains and nausea. Objectively, his
Krok 2- 2019 (General Medicine) (t.me/krok2GeneralMedicine) - 24

skin is gray; there is a pink-gray stripe on his gums; the gynaecological examination the uterus is unchanged.
stomach is soft and sharply painful. Blood test detected The appendages are bilaterally enlarged and painful.
erythrocytes with basophilic stippling and anemia. The Profuse purulent discharge is being produced from the
patient has a history of peptic ulcer disease of the vagina. What examination needs to be conducted to
stomach. Constipation tends to occur every 3-4 days. clarify the diagnosis?
What is the most likely provisional diagnosis? 1. Colposcopy
1. Saturnism (Lead poisoning) 2. Bacteriological and bacterioscopic analysis
2. Acute cholecystitis 3. Laparoscopy
3. Acute appendicitis 4. Curettage of the uterine cavity
4. Perforation of gastric ulcer 5. Hysteroscopy
5. Chronic alcoholism
183. A 59-year-old man complains of pain in his left eye
179. During regular preventive gynaecological and left side of his head, significant vision impairment of
examination a 30 year old woman was detected to have the left eye, nausea, and vomiting. Visual acuity of the
dark blue punctulated „perforations‟ on the vaginal right eye is 1.0. Visual acuity of the left eye is 0.03,
portion of the uterine cervix. The doctor suspects attempts at correction bring no improvement. Right eye
endometriosis of the vaginal portion of the uterine cervix. intraocular pressure – 21 mm Hg, left eye intraocular
What investigation method would be most informative for pressure – 65 mm Hg. Congestive injection is observed
diagnosis confirmation? on the sclera of the left eye. The cornea is thick and
1. US of the lesser pelvis swollen. The anterior chamber is shallow, moist, and
2. Hormone testing clear. The pupil is dilated and unresponsive to the light,
3. Curettage of the uterine cavity the fundus of the eye is not visible. What is the most
4. Hysteroscopy likely diagnosis?
5. Colposcopy, target biopsy of the cervix 1. Acute attack of glaucoma of the left eye
2. Panophthalmitis of the left eye
180. A middle school teacher with 4 year long record of 3. Endophthalmitis of the left eye
work was issued a medical certificate for pregnancy and 4. Stage II intraocular tumor of the left eye
childbirth leave. What amount of pay will she receive for 5. Acute iridocyclitis of the left eye
the duration of her leave in this case?
1. 60% of average salary 184. A 39-year-old man suffers from chronic rheumatic
2. 80% of average salary heart disease. He complains of dyspnea during physical
3. 100% of average salary exertion, cough with expectoration and palpitations.
4. 50% of average salary Auscultation detects intensified I heart sound and
5. 70% of average salary diastolic murmur; the sound of opening mitral value can
be auscultated at the cardiac apex. The II heart sound is
181. A 28-year-old man complains of skin rash and accentuated over the pulmonary artery. The patient is
itching on the both of his hands. The condition persists cyanotic, X-ray shows dilated pulmonary root and
for 1.5 years. The exacerbation of his condition he enlargement of the right ventricle and left atrium. What is
ascribes to the occupational contact with formaldehyde the most likely diagnosis?
resins. Objectively the lesion foci are symmetrically 1. Patent ductus arteriosus
localized on both hands. Against background of 2. Coarctation of the aorta
erythema with blurred margins there are papulae, 3. Mitral stenosis
vesicles, erosions, crusts and scales. What is the most 4. Pulmonary artery stenosis
likely pathology? 5. Aortic stenosis
1. Idiopathic eczema
2. Simple contact dermatitis 185. A 5-year-old child was brought to the ENT
3. Occupational eczema department by an ambulance. The child presents with
4. Allergic dermatitis cough and difficult respiration. From the patient‟s history
5. Erythema multiforme it is known that the child was playing with a toy
construction set, when suddenly started coughing and
182. A 30 year old woman came to the gynaecological developed labored breathing. Examination detects
department. She complains of sharp pain in her lower periodical cough, labored expiration and respiratory lag
abdomen and temperature of 38.8 o She has a history of in the left side of the child‟s thorax. Auscultation:
extramarital sexual activity and 2 artificial abortions. On diminished respiration on the left. Percussion:
Krok 2- 2019 (General Medicine) (t.me/krok2GeneralMedicine) - 25

tympanitis. X-ray shows a displacement of the 3. Venous


mediastinal organs to the right. Make the diagnosis: 4. Capillary
1. A foreign body in the right bronchus, partial 5. Mixed
bronchostenosis
2. A foreign body in the left bronchus, complete 189. A 48 year old woman has been hospitalized due to
bronchostenosis development of tachysystolic atrial fibrillation. She has
3. A foreign body in the right bronchus, valvular lost 5 kg of body weight within 2 months. On palpation,
bronchostenosis there is node in the left lobe of the thyroid gland. What
4. A foreign body in the left bronchus, valvular pathology resulted in the development of this condition?
bronchostenosis 1. Non toxic nodular goitre
5. A foreign body in the trachea 2. Autoimmune thyroiditis
3. Toxic nodular goitre
186. A 23 year old woman came to the gynaecological 4. Atherosclerotic cardiosclerosis
clinic. She complains of pain, itching, and burning in her 5. Chronic thyroiditis
vulva, general weakness, indisposition, elevated body
temperature upto 37.2 oC, and headache. On 190. A woman has been working as a polisher for a year
examination in the vulva there are multiple vesicles upto and half. Her workstation is equipped with a grinding
2-3 mm in diameter with clear contents against the machine (grinding wheels). She complains of white
background of hyperemia and mucosal edema. Make discoloration of her fingers and toes that appears when
the provisional diagnosis: she is nervous. Objectively there are no changes in the
1. Genital herpes infection coloration of the distal segments of her limbs. Grip
2. Primary syphilis strength measured with a dynamometer is 25kg,
3. Vulvar cancer algesimetry findings are 0.1; 0.3; 0.5. Cold stimulus is
4. Cytomegalovirus infection extremely positive on the upper and lower limbs. Internal
5. Papillomavirus infection organs are without pathogens. Make the diagnosis:
1. Raynaud syndrome
187. A 62-year-old woman was brought into the 2. Syringomyelia
admission room with complaints of severe burning 3. Polyneuritis
retrosternal pain and asphyxia. She has a 10 year long 4. Raynaud disease
history of essential hypertension. Objectively her 5. Vibration disease
condition is moderately severe. She presents with skin
pallor, cyanotic lips and vesicular respiration over her 191. A 26-year-old woman presents with amenorrhea.
lungs. The II heart sound is accentuated over the aorta. 10 months ago she gave birth for a second time. In her
Blood pressure – 210/120 mmHg, heart rate (pulse) – early postpartum period she developed a massive
76/min. ECG shows elevation of ST segment in the lead hypotonic haemorrhage. No breastfeeding. Lately she
I, AVL and V5-V6. What is the most likely diagnosis? has been presenting with loss of weight, loss of hair and
1. Hypertensive crisis complicated with instable angina indisposition. Gynecological examination revealed
pectoris atrophy of the external genitals, the uterus is abnormally
2. Hypertensive crisis complicated with acute left small, no uterine appendages can be detected. What is
ventricular failure the most likely diagnosis?
3. Uncomplicated hypertensive crisis 1. Sheehan syndrome (Postpartum pituitary gland
4. Pulmonary embolism necrosis)
5. Hypertensive crisis complicated with acute 2. Stein-Leventhal syndrome (Polycystic ovary
myocardial infarction syndrome)
3. Physiological amenorrhea
188. A surgery unit received a person with an incised 4. Suspected progressing ectopic pregnancy
stab wound on the upper third of the right thigh. 5. Galactorrhea-amenorrhea syndrome
Examination detects an incised stab wound 3.0×0.5×2.0
cm in size on the inner surface of the upper third of the 192. A prurient woman is 30-year-old, stage I of the
right thigh. Bright-red blood flows from deep within the labour is ongoing. The fetus is in the cephalic
wound in a pulsing stream. Characterize this type of presentation. Auscultation of the fetal heart sounds
bleeding: detects bradycardia. Evaluation of cardiotocogram
1. Arterial yielded the following date: decreased of basal heart rate
2. Parenchymatous down to 90/min, variability – monotonous (2 and less);
Krok 2- 2019 (General Medicine) (t.me/krok2GeneralMedicine) - 26

late decelerations with amplitude of 50/min. Make the Make the provisional diagnosis:
diagnosis and choose the obstetrical tactics necessary in 1. Tuberculous meningitis
this case: 2. Myelitis
1. Fetal distress. Forceps delivery 3. Chorea minor
2. Fetal distress. Vacuum extraction delivery 4. Brain tumor
3. Normal condition of the fetus. Vaginal birth. 5. Convexital arachnoiditis
4. Fetal distress. Urgent caesarean section delivery
5. Fetal distress. Stimulation of uterine contraction 196. A 27 year old woman, a teacher in the elementary
school, complains of frequent stools, upto 3 times per
193. A 34 year old man complains of pale edema of the day, with lumpy feces and large amount of mucus,
face, feet, shins and lumbar area, elevated BP upto abdominal pain that gradually abates after a defecation,
160/100 mmHg, and general weakness. He has a irritability. Her skin is pale and icteric. Pulse is 74/min.,
clinical history of nonspecific ulcerative colitis. rhythmic, can be characterized as satisfactory. Blood
Objectively: pulse – 84/min, rhythmic, blood pressure – pressure is 115/70 mmHg. The abdomen is soft,
165/100 mmHg; edemas all over the body; the skin is moderately tender along the colon on palpation.
pale and dry, with low turgor. The kidneys cannot be Fiberoptic colonoscopy detects no changes. What
palpated, on an attempt to palpate them they are disease can be suspected?
painless. Blood test: erythrocytes – 3.0 x 1012/L, Hb – 1. Irritable bowel syndrome
100 g/L, erythrocyte sedimentation rate – 50 mm/hour. 2. Crohn disease (regional enteritis)
Urinalysis: proteins – 3.5 g/L, erythrocytes – 7-10 in the 3. Chronic enteritis
vision field, leukocytes – 5-6 in the vision field. Daily 4. Chronic non-ulcerative colitis
proteinuria – 6 grams. What analysis should be 5. Whipple disease
conducted additionally to verify the diagnosis?
1. Radioisotopic examination of kidneys 197. A 3 year old child presents with dyspnea that
2. Renal ultrasound abates in the sitting position, occasional loss of
3. Survey and excretory urography consciousness and seizures, delayed physical
4. Gingival biopsy for the diagnosis of amyloid disease development, cyanosis, drumstick fingers.
5. Urinalysis for Bence-Jones protein Echocardioscopy detects aortic dextroposition,
ventricular septal defect, pulmonary artery stenosis, and
194. A 57 year old woman complains of weakness, right ventricular hypertrophy. What is the most likely
dyspnea, loss of appetite and liquid feces. She has been diagnosis?
suffering from this condition for 2 years. Objectively she 1. Acquired valvular disease
presents with pale skin, subicteric sclerae and bright red 2. Tetrad of Fallot
fissured tongue. Lymph nodes are not enlarged. Pulse – 3. Ventricular septal defect
100/min. BP – 105/70 mmHg. Liver +3cm, the spleen 4. Coarctation of the aorta
cannot be palpated. Blood test: erythrocytes – 1.2 x 5. Transposition of the great vessels
1012/L, Hb – 56 g/L, color index – 1.4, macrocytes,
leukocytes – 2.5 x 109/L, eosinophils – 1%, juvenile – 198. A 78 year old man with a prostate adenoma
1%, metamyelocytes – 1%, band neutrophils – 8%, underwent a herniotomy for a direct inguinal hernia. After
segmented neutrophils – 47%, lymphocytes – 38%, the surgery he presents with absent urination. Enlarged
monocytes – 4%, reticulocytes – 0.1%, platelets – 100 x urinary bladder is detectable above the patient‟s pubis.
109/L, ESR – 30 mm/hour, indirect bilirubin – 26 mmol/L. What measures should be taken in this case?
What changes can be expected in the bone marrow 1. Prescribe antispasmodics subcutaneously
puncture material? 2. Prescribe proserin (neostigmine) intramuscularly
1. Erythtroid hyperplasia 3. Prescribe processing of the postoperative wound with
2. Prevalence of megaloblasts UHF field
3. Increased number of sideroblasts 4. Apply cold to the urinary bladder area
4. Prevalence of lymphoid tissue 5. Bladder catheterization
5. Present of blast cells
199. A woman with atopic bronchial asthma was found
195. A patient is being treated in the tuberculosis clinic. to have one allergen to dog hair +++. Carpets are
Throughout the last 3 weeks he has been suffering from removed from the apartment, the apartment was
headaches of increasing intensity. Neurological renovated, and air conditioner was installed. However,
examination detects nuchal rigidity without focal signs. recurrent asphyxia attacks still occurs every night,
Krok 2- 2019 (General Medicine) (t.me/krok2GeneralMedicine) - 27

despite the patient undergoing pathogenetic therapy.


What long term treatment tactics can help this patient to
decrease her sensitivity to the allergen?
1. Buteryko breathing technique
2. Referral for speleotherapy
3. Specific hyposensitization
4. Antihistamine therapy
5. Continuation of prior treatment

200. 2 hours after eating unknown mushroom‟s, a 28-


year-old man sensed a decrease in his mobility and
deterioration of his ability to focus. This condition was
then followed by a state a agitated and aggression. On
examination he is disoriented, and his speech is illegible.
4 hours later he developed fetor hepaticus and lost his
consciousness. What syndrome can be observed in this
patient?
1. Acute hepatic failure
2. Hepatolienal syndrome
3. Cytolytic syndrome
4. Portal hypertension
5. Cholestatic syndrome
Krok 2- 2019 (General Medicine) (t.me/krok2GeneralMedicine) - 28

Krok 2 - 2019 (General Medicine) Answer Key


01.(3) 31.(2) 61.(1) 91.(3 121.(4 151.(2) 181.(3)
02.(2) 32.(1) 62.(5) 92.(1 122.(3 152.(2) 182.(2)
03.(3) 33.(4) 63.(5) 93.(2 123.(3 153.(1) 183.(1)
04.(1) 34.(1) 64.(3) 94.(1) 124.(1) 154.(4) 184.(3)
05.(5) 35.(5) 65.(4) 95.(3) 125.(2) 155.(3) 185.(4)
06.(5) 36.(4) 66.(2) 96.(5) 126.(4) 156.(2) 186.(1)
07.(5) 37.(5) 67.(5) 97.(3) 127.(3) 157.(1) 187.(5)
08.(3) 38.(2) 68.(3) 98.(3) 128.(3) 158.(2) 188.(1)
09.(5) 39.(4) 69.(4) 99.(2) 129.(3) 159.(3) 189.(3)
10.(5) 40.(2) 70.(5) 100.(2) 130.(1) 160.(4) 190.(5)
11.(4) 41.(3) 71.(3) 101.(2) 131.(2) 161.(2) 191.(1)
12.(1) 42.(3) 72.(4) 102.(5) 132.(5) 162.(4) 192.(4)
13.(4) 43.(4) 73.(1) 103.(4) 133.(3) 163.(3) 193.(4)
14.(3) 44.(1) 74.(4) 104.(1) 134.(3) 164.(2) 194.(2)
15.(3) 45.(2) 75.(1) 105.(5) 135.(2) 165.(2) 195.(1)
16.(4) 46.(3) 76.(2) 106.(1) 136.(1) 166.(2) 196.(1)
17.(3) 47.(5) 77.(1) 107.(1) 137.(1) 167.(5) 197.(2)
18.(1) 48.(3) 78.(2) 108.(2) 138.(2) 168.(1) 198.(5)
19.(4) 49.(3) 79.(4) 109.(3) 139.(5) 169.(2) 199.(3)
20.(4) 50.(3) 80.(2) 110.(1) 140.(3) 170.(3) 200.(1)
21.(2) 51.(5) 81.(2) 111.(1) 141.(3) 171.(3)
22.(3) 52.(2) 82.(5) 112.(1) 142.(4) 172.(4)
23.(5) 53.(3) 83.(4) 113.(1) 143.(5) 173.(2)
24.(3) 54.(5 84.(4) 114.(5) 144.(3) 174.(5)
25.(2) 55.(5) 85.(2) 115.(4) 145.(3) 175.(4)
26.(1) 56.(2) 86.(3) 116.(1) 146.(2) 176.(2)
27.(4) 57.(5) 87.(5) 117.(4) 147.(1) 177.(5)
28.(2) 58.(2) 88.(1) 118.(2) 148.(1) 178.(1)
29.(5) 59.(2) 89.(1) 119.(5) 149.(1) 179.(5)
30.(3) 60.(3) 90.(3) 120.(1) 150.(3) 180.(3)

You might also like